Full Book of CAIE Further Maths PDF

You might also like

Download as pdf
Download as pdf
You are on page 1of 646
Lee Mckelvey Martin Crozier Cambridge International AS &A Level Further Mathematics Coursebook & CAMBRIDGE UNIVERSITY PRESS CAMBRIDGE UNIVERSITY PRESS ‘University Printing House, Cambridge CB2 88S, United Kingdom ‘One Liberty Plaza, 20th Floor, New York, NY 10006, USA 477 Williamstown Road, Port Melbourne, VIC 3207, Australia 314-301, 3rd Floor, Plot 3, Splendor Forum, Jasola District Centre, New Delhi ~ 110025, India 79 Anson Road, #06-04/06, Singapore 079906 Cambridge University Press is part ofthe University of Cambridge, It furthers the University’s mission by disseminating knowledge in the pursuit of education, lesrning and research at the highest international levels of excellence, woweambridge.org Information on tis title: www.cambridge.org/9781108403375 ‘© Cambridge University Press 2018 ‘This publication isin copyright. Subject to statutory exception and tothe provisions of relevant collective licensing agreements, ‘no reproduction of any part may take place without the written permission of Cambridge University Press. First published 2018 2019 18 17 1615 1413 121) 19987654321 Printed in the United Kingdom by Latimer Trend A catalogue record for this publication is avaiable from the British Library ISBN 978-1-108-40337-5 Paperback (Cambridge University Press has no esponsibility for the persistence or accuracy (of URLs for external or third-party internet websites referred to in this publication, land does not guarantee that any content on such websites is, or will remain, accurate or appropriate, Information regarding prices, travel timetables, and other factual information given inthis work is correct atthe time of fist printing but ‘Cambridge University Press does not guarantee the aceuracy of such information thereafter. @IGCSE isa registered rademark Past exam paper questions throughout are reproduced by permission of Cambridge Assessment International Education. Cambridge Assessment International Edueation bears no responsibilty for the example answers to questions taken from its past question papers which are contained in this publication. ‘The questions example answers, marks awarded andlor comments that appear inthis book were written by the author(s). fn examination, the way marks would be awarded 10 answer like these may be different. NOTICE TO TEACHERS IN THE UK Itis illegal to reproduce any part of ths work in material form (including ‘photocopying and electronic storage) except under the following circumstances: {i) where you are abiding by a licence granted to your school ot institution by the Copyright Licensing Agency; (i) where no such licence exists, or where you wish to exceed the terms ofa licence, and you have gained the written permission of Cambridge University Press, (ii) where you are allowed to reproduce without permission under the provisions ‘of Chapter 3 of the Copyright, Designs and Patents Act 1988, which covers. for ‘example, the reproduction of short passages within certain types of educational {anthology and reproduction forthe purposes of setting examination questions. Contents Introduction How to use this book Acknowledgements Further Pure Mathematics 1 1 Roots of polynomial equations LI Quadratics 1.2. Cubies 13 Quartics 14 Substitutions End-of-chapter review exercise | 2 Rational functions 2.1 Vertical asymptotes 2.2 Oblique asymptotes 2.3 Inequalities 24 Relationships between curves End-of-chapter review exercise 2 3 Summation of series 3.1 The summation formulae Er, 3/2, E73 3.2 Converging series End-of-chapter review exercise 3 4 Matrices 1 4.1. Matrix operations 4.2 The inverse matrix 4.3 Determinants 44 Matrix transformations End-of-chapter review exercise 4 wou ee 10 15 16 24 ERB 45 BT 37 58 59 65 nD 16 92 ‘Cambridge International AS & A Level Further TEU 5 Polar coordinates 5.1 The polar system 5.2. Applications of polar coordinates End-of-chapter review exercise 5 6 Vectors 6.1 The vector product rule 6.2 Vector equation of a line 6.3 Planes End-of-chapter review exercise 6 7 Proof by induction 7.1 The inductive process 7.2. Proof by induction for divisibility End-of-chapter review exercise 7 Cross-topic review exercise 1 Further Probability & Statistics 8 Continuous random variables 8.1 The probability density function 8.2. The cumulative distribution funetion 8.3 Calculating E(g(¥)) for a continuous random variable 8.4. Finding the probability density function and cumulative distribution function of ¥= g(¥) End-of-chapter review exercise 8 9 Inferential statistics 9.1 t-distribution 9.2. Hypothesis tests concerning the difference in means 93. Paired t-tests 9.4 Confidence intervals for the mean of a small sample 9.5. Confidence intervals for the difference in means End-of-chapter review exercise 9 10 Chi- 10.1 Forming hypotheses quared tests 10.2 Goodness of fit for discrete distributions 3 103, 47 118 119 123 128 138 139 140 146 151 152 154 155 161 174 178 188 189 190 197 203 207 210 219 220 221 227 10.3 Goodness of fit for continuous distributions 10.4 Testing association through contingency tables End-of-chapter review exercise 10 11 Non-parametric tests 11.1 Non-parametric tests 11.2 Single-sample sign test 11.3 Single-sample Wilcoxon signed-rank test 11.4 Paired-sample sign test, 115 Wilcoxon matched-pairs signed-rank test, 11.6 Wilcoxon rank-sum test End-of-chapter review exercise 11 12 Probability generating functions 12.1 The probability generating function 12.2 Mean (E(X)) and variance (Var(¥)) using the probability generating function 12.3 The sum of independent random variables 12.4 Three or more random variables End-of-chapter review exercise 12 Cross-topic review exercise 2 Further Mechanics 13 Project 13.1 Motion in the vertical plane s 13.2 The Cartesian equation of the trajectory End-of-chapter review exercise 13 14 Equilibrium of a rigid body 14.1 The moment of a force 14.2 Centres of mass of rods and laminas 14.3 Centres of mass of solids 144 Objects in equilibrium End-of-chapter review exercise 14 231 237 248 249 250 251 254 260 263 266 2n7 279 280, 287 292 298 304 305 308 309 314 320 321 322 326 336 341 352 rn NCE PVN a auc} 15 Circular motion 15.1 Horizontal circles 15.2 The 3-dimensional case 15.3 Vertical circles End-of-chapter review exercise 15 16 Hooke’s law 16.1 Hooke’s law 16.2 Elastic potential energy 16.3 The work-energy principle End-of-chapter review exercise 16 17 Linear motion under a variable force 17.1 Acceleration with respect to time 17.2 Acceleration with respect to displacement End-of-chapter review exercise 17 18 Momentum 18.1 Impulse and the conservation of momentum 18.2 Oblique collisions and other examples End-of-chapter review exercise 18, Cross-topic review exercise 3 Further Pure Mathematics 2 19 Hyperbolic functions 19.1 Exponential forms of hyperbolic functions 19.2 Hyperbolic identities 19.3 Inverse hyperbolic functions 19.4 Logarithmic form for inverse hyperbolic fun End-of-chapter review exercise 19 20 Mat 20.1 Eigenvalues and eigenvectors es 2 20.2 Matrix algebra 20,3 Diagonalisation 204 Systems of equations End-of-chapter review exercise 20 397 398 403 409 410 4 a7 429 430 431 432 438 440 442 446 447 448 451 461 465 an 21 Differentiation 472 21.1 Implicit functions 473 21.2 Parametric equations 478 21.3 Hyperbolic and inverse functions 481 21.4 Maclaurin series 488 End-of-chapter review exercise 21 496 22 Integration 497 22.1 Integration techniques 498 22.2 Reduction formulae 503 22.3 Arc length and surface areas 509 22.4 Limits of areas 515 End-of-chapter review exercise 22 523 23 Complex numbers 524 23.1 de Moivre’s theorem 525 23.2 Powers of sine and cosine 529 23.3 The roots of unity 532 go 23.4 Complex summations 5337 End-of-chapter review exercise 23 543 24 Differential equations 544 24.1 First order differential equations 545 24.2 Second order differential equations: The homogencous case 550 24.3 Second order differential equations: The inhomogeneous case 557 24,4 Substitution methods for differential equations 565 End-of-chapter review exercise 24 575 Cross-topic review exercise 4 576 Further Pure Mathematics 1 practice exam-style paper 578 Further Probability & Statistics practice exam-style paper 579 Further Mechanics practice exam-style paper 581 Further Pure Mathematics 2 practice exam-style paper 582 Cambridge International AS & A Level Further h The standard normal distribution function Answers: Glossary Index 629 tec eat Introduction Cambridge International AS & A Level Further Mathematics is a very rigorous and rewarding course that builds ‘on the A Level Mathematics course, The Further Mathematics course is designed for students who wish to understand mathematics at a much higher level, and who have already successfully completed the A Level Mathematics course. With careful planning, Further Mathematics can be studied alongside A Level Mathematics. The course is divided into three major areas: Pure Mathematics, Probability & Statistics and Mechanics. There are 13 Pure Mathematics topics, 5 Probability & Statistics topics and 6 Mechanics topics, which make up the four examination papers that are available to students. Due to the flexible nature of the modules, students can take either AS Further Mathematics ot A Level Further Mathematics. The 24 topics build on knowledge already acquired in the A Level Mathematies course This coursebook has been written to reflect the rigour and flexibility of the Further Mathematics course. The authors have almost 30 years of Further Mathematics teaching experience between them, and have used their experience to create a comprehensive and supportive companion to the course. While the majority of the examples are within the scope of the course, there are opportunities for discussion and examples that will stretch the curious mind. The book is designed not only to instruct students in what is required, but also to help them develop their own. understanding of important concepts, Frequent worked examples guide students through the steps in a solution, Numerous practice questions and past paper questions provide opportunities for students to apply their learning, go In addition, there are cross-topic review exercises and practice exam-style papers for students to consolidate what they have covered during the course. The questions have been written to provide a rich and diverse approach to solving problems with the intention of enhancing deep learning, Every care has been taken to ensure that the English used in this book is accessible to students with English as an additional language. This is supported by a glossary of the key terms that are essential to the course. ‘The authors wish you the very best as you embark on this course Lee Mckelvey Martin Crozier Past exam paper questions throughout are reproduced by permission of Cambridge Assessment International Education, (Cambridge Assessment International Education bears no responsibility for the example answers to questions taken from its past question papers that are contained in this publication. ‘The questions, example answers, marks awarded andlor comments that appear in this book were writen by the author(s). In an examination, the method in which marks would be awarded to answers similar to these may be different. How to use this book Throughout this book you will notice particular features that are designed to help your learning. This section provides a brief overview of these features. aenneene tone Treen f Learning objectives indicate the cone nportant ts within each chapter and help you to fe through the coursebook Prerequisite knowledge exercises identify prior learning that you need to have covered before starting the chapt Try the questions to identify any areas that you need to review before continuing with the chapter. Key point boxes contain a summary of the most importa tsand In this section we will be looking at cubic equations, : N\ are », Gola aease Key terms are important terms in the topic that you ese areleaming, They ae highlighted in orange bold. The * glossary contain: ar definitions of these key terms Worked examples provide step-by-step approaches to answering questions. The left side shows a worked x= solution, while the right side contains a commentary f explaining each step in the working, | consis nim tempos having cote Oand aa with amar Explore boxes ci work, Thess tain enrichment activities for extension Note the citer activities promote group-work and peer er discussion, and are intended to deepen your standing of a concept between (2a)? and Tip boxes contain helofu guidance about calculating or checking your answ CCS UL tg Did you know? boxes contain interesting facts showing how Mathematics relates to the wider world Checklist of learning and understanding # Srabanen x At the end of each chapter there is a Checklist of learning and understanding, Ihe checklist contains a summaty of the concepts that were covered in the chapter. Youcan use this to quickly check that you have covered the main topics. 1 cena A= (58) mee (4 find Couch that BC= A. A? determine D, where ADI \ ‘The End-of-chapter review contains exam-style questions covering al topicsiin the chapter. You can use this to check your understanding ofthe topics you have covered Cross-topic review exercises appear after several chapters, and cover topics from across the preceding chapters cmos 2 exe mstemate natn at Nee serine menor Y Orem OES Recall from Chapter 15 that, for particles You will meet these formulae again in Chapter 7, when you will prove them in a more rigorous way travelling in horizontal circles, the tension component is directed towards the centre Rewind and Fast forward boxes direct you to related learning, Rewind boxes refer to earlier learning, in case you need to revise a topic. Fast forward boxes refer to topics that you will cover at a later stage, in case you would like to extend your study. ~ Extension material goes beyond the syllabus. Itis highlighted by a red tine to the left of the text. Throughout each chapter there are multiple ‘exercises containing practice questions, The ‘questions are coded ‘These questions focus on problem-solving. These questions focus on proots. These questions focus on modelling, You should nat use a calculator for these questions, You can use a calculator for these questions, ©e@ 9se08e These questions are taken from past examination papers. etn Me ga Eo Acknowledgements The authors and publishers acknowledge the following sources of copyright material and are grateful for the permissions granted. While every effort has been made, it has not alvays heen possible to identify the sources of all the ‘material used, or to trace all copyright holders. If any omissions are brought to our notice, we will be happy to include the appropriate acknowledgements on reprinting. Past exam paper questions throughout are reproduced by permis Education ion of Cambridge Assessment International Cambridge Assessment International Education bears no responsibility for the example answers to questions taken from its past question papers that are contained in this publication, Thanks to the following for permission to reproduce the images: Cover image darios44/Getty Images Inside (in order of appearance) 8. Lowry/Univ UlsteriGetty Images, John Crouch/Getty Images, franckreporter! Getty Images, Ann Monn/Getty Images, lan Hobson/Getty Images, Paul Bradbury/Getty Images, Derek Bacon/ Getty Images, Mitchell Funk/Getty Images, Martin Barraud/Getty Images, Aidan Richards/EyeEm/Getty Images, Frederic Cirou/Getty Images, Jay's photo/Getty Images, t_kimura/Getty Images, Tawan Prakaisakul/ Bi EyeEm/Getty Images, Ken Reid/Getty Images, MirageC/Getty Images, tobishinobi/Getty Images, Michelle Pedone/Getty Images, Matt Nolan/EyeEm/Getty Images, Andy Crawford/Getty Images, joe daniel price/Getty Images, Frank KrahmeriGetty Images, Derrick Argent Photography/Getty Images, John Thurm/EycEm/Getty Images, Richard Kail/Getty Images ee eR ee cua ea Lue Senora Where it comes from What you should be able todo Check your skills, ‘AS & A Level Mathematics | Use simple substitutions to. | 1_Rewrite the following equations in terms Pure Mathematics 1, | make another variable the of the new variable. Chapter 2 subject. a aa es b e+27- © 8-3x47=0,y ‘AS & A Level Mathematics | Work with basic sigma Evaluate the following Probability & Statistics I, | notation, such as Ex and Ex? 10 Chapter 2 ‘AS & A Level Mathematics | Work with basic recurrence Write the first six terms for the following Pure Mathematics 1, | relations, relations Chapter 6 A tiyyt = Bip + 2s = 1 Bugg = tps ~ te + Sth = Lotte What are polynomials? | Polynomials are algebraic expressions made up of one or more variables and a sum of terms involving non-negative integer powers of variables. For example, 2x* ~ 3xy + Sx is splynonial btreiter et norare polnoi ngats epost that a new building can withstand the force of an earthquake. Medical researchers use them to model the behiaviour of bacterial colonies. We already know how to divide a polynomial by a linear term and identify the quotient and any remainder, We have worked with simpler polynomials when completing the square of a quadratic o finding the discriminant. Now we will extend this knowledge to work with higher powers. We will also use algebraic manipulation to understand the conditions for complex solutions and to combine polynomials with summation notation and recurrence relations, In this chapter, we will look at ways to find characteristics of polynomials, finding the sum and product of roots as well as other properties linked to their roots. 1.1 Quadratics To begin with, let us look back at the quadratic equation ax? +x + If we write this inthe form 3+ Se £0, thn we can compare iio the form (s=a)t~f)= 0. This shows that tbesum fhe rots + shown in Key point 1.1. Hence, we can say that x? — (a+ f)x + af = 0. 8 atthe potuct ofthe oo oP = 98 a ep eS Nae eee pees ote ee arene oe ), the sum of the rootsis «+ ‘The product of the roots of the quadratic equation is a Peteicuaiss ‘The quadratic equation x? — 2px + p =O is such that one root is three times the value of the other root. Find p. Answer a+3a=2p ? ax3a=p Using ap = 5 pa Equate the two results, ‘Cross multiply. genbk 8 gi : Factorise and omit the case when > | 4, we can begin to define many other results, but first we must introduce some new notation. The sum of the roots can be written as Za =a +f and the product can be written as Zap = ap. Let us consider how to determine the value of a? + f®. The natural first step isto expand (+f) =a? +p +2ap, Hence, we can say that «2+ p= (Sa)? ~ 2Eay. We denote OB as Eee ‘Note the difference between (Za)* and Sa Next, look at (a) Again, expanding the brackets isa good start. So (a—p)? =a? + ~ 2ap. Hence, we can see that (a — f)* = Za? ~ 2Eap. Weean wie Le 2h How dowetndctesumet 1+ I Ft combine the vo 1 ee ectonstost “2, Wecanas thats is 2 Sintly wean wri Leg! +121 on of Pica we can show ery Pee ROS ee tua ea a CUS g cients es Find o + f° in summation notation. Answer (a4 BP =a? + 3a + 30? + ‘Use the binomial expansion for (x-+ 5)" +P =(a+ 9) —3apla +p) Rearrange and factorise. Za = (Ba) - 32afZa ‘Sum for each possible root. Alternatively, use However, it is not as easy to calculate with this form, Some of the results found can be written in alternative forms, using a recurrence relation such as S, =a" +9". If we consider the quadratic equation x7 + 5x + 7=0, we can see that =5. This result can also be viewed as Sy =a + = ~5. To determine the value of +p, we can approach this from another angle. Given that @ and f are roots of the original equation, we can state that a? + Sa+7= and §? + 58+ 7-= 0. Adding these together gives the result (« + f°) + S(@ + A) +14=00r 'S) +55, + 14=0, Now we can work out the value of S, or a? + 6°. From S> + 5S) + 14=0 and 5 we have S3= a? + f= I1. Note this could also have been found from 20) Peciaruutsey Given that 2x2 + 3x —2= 0 has roots a. f, find the values of a? + f* and a® +p. Answer 2a? +3a—2=0 2p 4 3p-2=0 35;-4=0 Add the two equations to get the recurrence form. State $, = ~8 from the original quadratic equation. Qae+ P= 7 Substitute the S; value into the equation. 2x? + 3x = 32x43 -2v=0 Multiply by ». 25,438, - 25 =0 ‘Add 2a} + 342-20 =0 and 26° + 39" — 29 Use the values of S; and $3. eek eeu Saesty 1 Each of the following quadratic equations has roots a, f. Find the values of a +f and af. a 45x49=0 b x 4x48=0 © Qx?43x-7=0 2 Given that 3x? +4x + 12=0 has roots a, §, find: a atfand af b rtp 3. x2—(2+p)x+ (7+p)=0 has roots that differ by 1. Find the value of p given that p> 0. @ 4 tae b=-3 and += 7, find the value of ab and, hence, write down a quadratic equation with roots aandb © 5 We +bx+e=0 as roots and p, prove tha 16, if a= 49, then = 18 a Be 3 b if a=f-2, then = 4(e+ 1), equation px? + gx ~ 16=0, whick has roots «and f, Given also that a+ p=—! @® 6 Youaresiven the quad and af = -8, find the values of p and g 7 The quadratic equation x7 +2x~6=0 has roots @ and f. Find the values of (a — f)? and ° - @ 8 A quadratic equation bas roots «and p. Given that £+4=1 and a? +p = 12, find two possible quadratic eaiations that satisfy these vals, 5 9. The quadratic equation 32? + 2x~4=0 has roots and f. Find the values ofS}, Sy and S. @ 10 Vouare given the quadratic equation 4x — x +6 =0 which has roots «and f. a Find + b Without solving the quadratic equation, state what your value for part a tells you about the roots. 1.2 Cubics In this section we will be looking at cubic equations, We will use the same concepts as in Section 1.1, but this time the roots will be a, and y. Following on ftom te ‘idea you sew in Worked Beginning with ax’ + bx° + ex + d=0, the first step is to divide by the constant a to get example 1.3, if we d consider the notation eebte see Send + p47 and then use itt represent ur roots just as with P= (4 P+) + (op 4ay + fy)x~ apy =0 ‘quadratic equations, we b can se S, to represent Next, relate this to (x — a(x ~ f(x = 7) = 0 to establish the relation: Then a+ p+y=—%, whichis known as Ea of 5 24247 and soon. 4 written as Zap Other results are ap + ay + fy = &, written as Zap, and apy Recall from quadraties that Ba? = (Za)? ~ 2Eaf. This is the same result for a cubic equation, where the term (Za)?= (a+ + y= a" + p+ 7° + 2af + 2ay + 26, as shown in Key point 1.2 Tea Uy Prony rag ean aE rere Cay Prins Find he summation frm or berenas Lt +P +P + 2a + Day +a+ Answer 11a eee ‘Combine the fractions. ag or a = State the result, Say ap eaie+ Pe a Combine the fractions, as before J # el = State the result a appt All ofthe results derived for quadratic equations can also be written for cubics, but the algebra is more complicated. Try to convince yourself that for a cubic itis true that Ea! = (Za)? - 3ZapEa + 3Eapr. reciente Given that x? + 2x? + 5 =0, find, using summation form, the values of S83, $3 and S_ Answer Si Recall thisis S= (Za)? ~2Bap -2x0=4 Recall the vatue of Bah is given by £=9 asthe linear term coefiicient is 0. i y= Ba)! ~ apa + Bap ‘The lst term is Say. = 3 x (0) x (-2) +3 x (-5) Substitute the values into the equation. Recall that this result is equivalent to which is obtained by taking the negative of the coefficient of the linear term and dividing by the constant term, Worked example 1.5 uses the summation form, but there is a more efficient way of finding Ss and higher powers, In Worked example 1.6 we will use the recurrence form to evaluate results such as S; and Ss. Consider the equation 2° + 3x" 4 6 = 0, Since a, f, yall satisly our cubics, we can see that 43a? + 6=0,f +36 +6=Oand? + 37°+6=0. ‘Adding the three equations gives «+ 84+ +4a2+ 6+ 7) +1 or S3+ 35) +18 =0. naan For the cubic equation 3x? + 2x? —4x+1=0, find the value of Sy. 35; +25,—45, +3 =0 peuene + 107 21 We have already seen how to manipulate a polynomial to get a higher power result, such as obtaining S; from a quadratic equation. Imagine we want to obtain a value such as S_p from a cubic equation, using only recurrence methods. 4 =0.The recurrence formula would then be aS; +38 + ¢S_y + dS_ = 0. Note the constant term, b, is multiplied by 3. Now we need to find only 5 and Sy, and from the original equation this is straightforward, ‘The first step would be to multiply our cubic by x~* to give ax + b+ E+ inated For the cubic equation 1° — 3x? + 4 = 0, find the value of Ss, Answer Pe Cu en RTC) en ea a We can generalise from Worked example 1.7, For @ general cubic of the form ax! + bx? + ex-+d= 0, if we multiply by x" then our recurrence formula is 4S,43 + bSqu2-+ Sys + 45; = 0. Note that only constant terms get counted multiple times, 1 Each of the following cubic equations has roots a, f 7. Find, for each case, a+f+y and afy. a 43-5 b 2x74 5x7-6=0 ¢ Y4Ix-9=0 2. Given that x? 3x? + 12=0 has roots a, 17 find the following values: a a+fty and af +ay+ by b Bae 3. The roots of each of the following cubic equations are a, fy. In each case, find the values of S; and S_, b 3044 € P43x745x-7=0 x-+7=0 has roots a, 9, y. Find the values of Za and Ya", a 2245 4 The cubic equation x7 © 5 Given that 22 + 5x? +1=0 has roots @,f,7, and that S,=a" + p" +’, find the values of S; and Ss. O@ « Thecubicequation P+ art beta 2: Find in tems andthe vals of Ea and 3 has roots «, f,7, and the constants a, bare real and positive. b Given that 5 O® 7 Treeabicequation 2x43 a Using the relation S, = a! + B+ or otherwise, find the value of Sy 0 has roots a,A,7. b_ Byconsidering S; and S,, determine the value of (f+) +Pa+1)+Pla+P) © & Acubic polynomials given as 2x! — 38+. ~ $= 0, having roots a. pur a Show that 28,43 — Sy42 + Suet — 58, = 0. b Find the value of S., © 9 Theeabicequation px) + gx as bs, cS has roots a, f, 7. Find, in terms of p, q.1: 10 The equation x'+ px? + gx-+r=0 is such that S) =0, Sp -2 and S.y=4, 5 Find the values of the constants p, 4, 1.3 Quartics Now that we are working with quartics, it s best to use the recurrence formula whenever wwe can. This is especially true for the sum of the cubes (a! + 6 + 7° +53). If we want to determine the sum of the cubes of a general quartic, the best way is to first note down 5}, then determine 5, and S_1. After this, we can use the form aS, + 6S: + 052 + dS + 4e = 0, then divide by x to obtain Sy, This process allows us to work out other values, especially those beyond the highest power. As we have seen with previous polynomials, there are standard results that are defined by observation from previous cases, but the algebra for some results is too complicated to be discussed here. So, with our roots a, 9, 7,5, we have Ea = ~5, Fa = 5, Sap We also have 5; = (2a)? — 28af and S_ and soon. Lay Lapis Algebraically itis much more sensible to use S,= a" + p" +1" +8" When converting a polynomial to & recurrence formula, the constant is always multiplied by n from the original equation. As an example, x‘ ~ 3x°- 5 =0 would give Sy 38; = 200. Eto Pry A quartic polynomial is given as x* + 3x7 —x+ 5 =O and has roots a, 6,7, 6. Find the values of Sand Sy. Remember that for any polynomial, 24 is aways obtained using Answer the negative ofthe coefficient of the lineer term over the constant term, Si=0 Ene For the quartic x4 x3 + 2x? 2v ~5=0, state the values of S, and S_,, and determine the value of S,. State whether or not there are any complex solutions. Answer S=1 Don't ry to use an algebraic approach for quartis, especially for ‘Syand higher. Use the recurrence method, 1 For each of the following quartic equations, find the values of Za and Zap. a x20 45x°4+7=0 b 2xt+ 5x3 3x4+4=0 © 3x4 2x74 9x-11=0 fener een E Tes 6@e @es QO: 1.4 Substitutions Imagine that we are given the quadratic equation x? + 3x-+ asked to find a quadratic that has roots 2a, 28. There are two approaches that we can take, First, consider the quadratic (y ~ 2a)(y ~ 2) ‘we compare this with the original, which is a + the new quadratic. This method requires us to know some results, or at least spend time working them out. A second method is to start with y = 2x, since each root of y is twice that of x. Then, substituting multiplying by 4, y+ 6 + 20 = 0, This second approach does not need the values of roots. Itjust needs the relationship between the roots of each polynomial frocianiateee ‘The roots of the quartic x*~ 2x? + auc Tra gees The quartic equation 5x4 - 3x3 + x - 13=0 has roots a, f, 7, 6. Find: a 2a and Ea? b xt A quartic equation is given as x4 + x+2=0. Ithas roots a f, 7,5, State the values of S; and S_, and find the value of Ss The quartic equation 2x! + x3 — of Sy +70 has roots a,f.7,5.Given that S,="1, and using S,, find the value You are given that xt 1° +x-+2=0, where the roots are a, 9,7, 8. Find the values of Ya, Za? and Et. Hence, determine the value of Br The quartic polynomial x4 + ax? + bx + 1 =0 has roots a, 6. Given that Sz = S_y, find S; in terms of a The polynomial 3x4 + 2x3 + 7x7 +4=0 has roots «,f,7,5, where S,=a" +p" + 7" +0". a. Find the values of S, and S;, b Find the values of 53 and Sy, ¢ Are there any complex roots? Give a reason for your answer. For the polynomial x! + ax} +.6x2 + ¢=0, with roots, p, yand 8, itis given that a+ P+ y+ 8=2, afyS=1 and a? +? +? +6’ =0. Find the values of the coefficients @, b and c are a, B,¥.6. Show that Sy =9Ss, 0 with roots a, # and we are You learned in AS & A Level Mathematics Pure Mathematics 1 Coursebook how to find inverse functions by interchanging xand y. The same process is helpful here 0, then y?— (2+ 2p)y + 4ap 3,af= 5, then y? + 6y +2 ir dis into the original gives (2)°+3(2) +5=0. Alternative nother! es (2)'+4(2) ' hhas roots ,f, find the quadratic equation with roots, Rearrange to make x the subject. Substitute for x. ‘Multiply out terms and simplify. erecta NaC More complicated substitutions include reciprocal functions. For example, consider the cubic function x° + x7—7=0 with roots af, 7. If we are asked to find a cubic Function apr 1D 4 (L)-7=0, which 2 (i) (I) 70m simplifies to the eubie 7)? — y ~ Eons Eroatsets hhas roots «yf, 7, find the cubic equation with roots with roots 1.4.1, we would begin with y Given that 2» +x? - a-Pp-Py-2 ‘ warmer ex the subject (-C) 14 6y +122 +8) Le dy t4y? x= rr ip ae a+ 1 ay +1 a yb of another cubic, what are the conditions on a and & to ensure that these cubies are the same? are the roots ‘The polynomial x° + x—3=0 has roots a, , . I Powers of roots require a different method. For example, if we have the cubic equation 2x} + 7x? — 1 = 0 with roots a, f,7 and we want to determine the cubic with roots a”, f°, 7°, there are two ways of approaching this. 3 First, we could state that y=? andso x= Vj Substituting gives 22 +7» — =2y? and square both sides, giving 49y? ~ I4y +1 =4y°. $0 Next, write as 7) — ay) —a9y2 + Lay — In the second approach we first rearrange the cubic to 2x3 = 1 — 7x7. Doing this allows us to square both sides and get even powers of x for every term, so 4x8 = 1 ~ 14x? + 494 Substituting in y gives the same cubic as before. 0 is the cubic that we are looking for. In the first approach we substitute y = x” before rearranging the equation. In the second approach we do the steps in the reverse order. For both approaches we need to make sure the powers of x are appropriate Procionuatsee! The polynomial 44.3 x-+ 12 Answer State the substitution, even terms. x4 2de4 4 1d = x? = 2 Square both sides, x8 + 26x4— 32+ 144=0 Simplify. oP +267 y+ 1440 Usex?=y. has roots a, fy, 6. Find the polynomial with roots a, ‘Rearrange so that both sides when squared give PE ed Cue eee a8. ‘These substitution methods are useful when dealing with problems such as finding the value of S; or even Sy. Consider the quartic x!+ x" — $=. For this polynomial, we would like to determine the value ofS, The process for finding S_= a" + f+ 7 + 6 can be time consuming. Now, consider that there is another quartic such that y = x*. If this quartic exists, then for y we would have S, =o" ++ 72" + 54. Since we have doubled the power for each root, once wwe have determined the quartic for » we only need to find Ss, which is straightforward ‘Rewrite the original quartic as x* Nest, replace x with yso that 3 and S,=1? 5, then square both sides to get x8 — 10x! 4 25 = x4 y8— 10)? +25 = 0. Finally, for the new quartic, Sy = 1 2.x (-10) =21. Hence, for the original quartic, Sy = 21 ‘This is an effective method and can save lots of time, particularly for much higher values of n Prciocuatsee) The cubic polynomial x + 5x +1 find the value of Si, has roots a, f, 7. Using the substitution +5241=0 () > Label the original equation. 4 1a—s2 Since we are using x, ensure all terms are atranged so when both sides ate ‘cubed, they produce powers that are multiples of 3. P+ 3x 43x) + 1 = —125x4 Cube both sides. 3x94 128x437 41 Simplity. Sy + 1? 43+ 0 2) », label the new equation. Ensute both sides of the rearranged polynomial will give appropriate powers when the squaring or cubing operation has taken place, For example, S-Sx+7= y=22 would be wwrittenas 2° - 5; to ensuce that, when squared, both sides produce only even powers If the same equation isused with y=.°, then rearrange 10 Sx so that, ‘when its cubed, the powers of x are ‘multiples of 3 on both sides. apa rte ce 5\=-128 (=128)? — 2 x 3 = 16378 for 2) Hence, for (I), Ss = 16378 Eon ‘The term ‘polynomials’ was not used until the 17th century. Before the 15th century, equations were represented by words, not symbols. A famous Chinese algebraic problem was written: “Three bundles of good crop, two bundles of mediocre crop, and one bundle of bad crop are sold for 29 dou’ In modern times we would phrase this as 3a+2h + e=29, Grete) has roots a, f. Find the quadratic equation with roots 3a, 36. © 1 The quadratic equation 2° + Sx-+ © 2 Thequadratic equation 22 - 4x +7=Ohas roots a, p. a Find the quadratic equation with roots a2, 2. Find the quadratic equation with roots 2a ~ 3, 26 — 3. mel et Be 3. Given that 3x7-2v.+9=0 has roots a, f, find the quadratic equation with roots @* 1, The qudemiceqnton 21— 4-490 haroetr ai Find ke ana ht hare 5. Given that 2x ~ 5x +1=0 has roots a, f,y, find the eubic equation with roots a, 62,7. Hence, find the value of Sy @® @6e°e 6 Thecubie equation x? + 3x? ~ 1=0 has roots «, f,7. Show that the cubic equation with roots a+2 p42 742 ap 7 (2428+ 27+2) pee ar afr a2” p42 742 O® 7 Aauartic equation, 2x4 — x — 6 =0, has roots a,f, 7.5. Show that the quartic equation with roots a?, f°, 75° is 8y*—y* — 18)? — 108y — 216 = 0. Hence, find the values of Sand 5. is y)- 3)? - 9y + 3 = 0. Hence, determine the values of: The cubic equation x3— x +4 =0 has roots a, 6, y. Find the cubic equation that has roots a”, 2, ?. Hence, or otherwise, determine the values of Se, Sy and So 2) er amen Te Luar ee a Aue oak eu) ‘Theequation 334 has roots a, 6,7 Show that the equation with roots «?, pis »2 ~ 3)2-+49— ‘Hence, find the value of af + f° + 7°. Cambridge International AS & A Level Further Mathematics 9231 Paper 1 Q5 June 2008 Answer Start with x4 ‘Then rewrite this as (x? ~ 1)? = —a°. This gives x? - 3x54 4x3 Let y= 3 to give » Note that S, =a" +" +7" -2x4 Sq for the original equation is S) for the new equation, so Sy Hence, a + f° + 7° ele ed CCU UCU eu LL 4 For quadratic equations (ax? + bx + ¢= 0: For cubic equations (ax* + bx? + ex + d=0): Ba=arpry=—> Lap = af + ar + Br= = apy = apy =—4 Sapte For quartic equations (ax! + bx? + ex? + dx +e= 0): Zanarptyto=—% Sap = af +07 + ab + fr f+ 76—£ Ey = apy + 00+ arb + Bro = 4 apd = apo == Saas payee For recurrence notation: © Beis also known ar S, © Ea! = (Ba)? ~ 22a isalso known as Sp © 2! ianounas Snaps ego henge of hecoeisint th int die by the eect of he coin om Chapter 1: Roots of polynomial equations @. munauadas 1 The roots of the equation x° + 4x 1=0 area, and y. Use the substitution y is to show that the ian 1 at B+ yel Tega i +l" B+ G+" 0 has roots equation 6)? ~ Ty? + 3y— a+ For the cases m= 1 and » = 2, find the value of Deduce the value of —!_4—1_,_1 @+)) GH o+P G+ DO+) G+ Dar) @+DG+D_73 Hence show that @+h? +i G+ 36 Cambridge International AS & A Level Further Mathematics 9231 Paper 1 Q7 November 2010 The roots of the quartic equation x* + 4x? 42x? — 4x +1=0 are a,f,yand 6, Find the values of i atpry+s, i daperee, 2 aby Using the substitution y = + 1, find a quartic equation in y. Solve this quartic equation and hence find the roots of the equation x* + 4x° + 2x? - 4x 41 = Cambridge International AS & A Level Further Mathematics 9231 Paper 11 Q1I November 2014 ‘The cubic equation x ~ x? — 3x— 10 =0 has roots «, 8,7. i Let w=~a++y. Show that w+ 2a= 1, and hence find a cubie equation having roots at ptya-p+yatp~y Cambridge International AS & A Level Further Mathematics 9231 Paper 13 Q8 June 2012 Rational functions WANN. a Pa CeCe Tat} Co area Where it comes from What you should be able todo | Check your skills 1S & A Leet Mathcmaizs "| Sl funeonsiat paral] Spl the llwing io pata acon Pure Mathematics? & 3, | fractions. | ead Chapter 7 a p —1 (-D@-2) AS & A Level Mathematics | Know about asymptotes and 2. Write down the asymptotes for: Pure Mathematics 1, how to determine horizontal and Chapter 5 | linear cases. | Work with the modulus function Pure Mathematics 1, in the forms y= |f(3)| and ‘Chapter 5 y= fx). AS & A Level Mathematics, ay by Pure Mathematics 2 & 3, ea Chapter 1 | What are rational functions? ‘A rational function is any funetion that can be defined as an algebraic fraction with polynomials as numerator and denominator, Rational functions are used to predict. ‘outcomes, from international trade balances in Economics to the amount of anaesthetic a patient will need for surgery. You are already familiar with plotting graphs as functions and identifying simple asymptotes, You should already have the algebraic skills to use partial fractions to rearrange and simplify functions. In this chapter, you will sketch several different types of function, including Y= [flo] and y= fils). You will also work with asymptotes 1 ity" (horizontal, vertical and oblique); identify symmetries of curves and similarities between ‘curves; and determine ranges, turning points and x-intercepts and y-intercepts. 2.1 Vertical asymptotes ‘An asympiote is generally a line that a curve approaches but does not touch. ‘An asymptote is generally a line that a curve approaches but does not touch, We can say the curve ‘converges £0 an asymptote, 3. Make sketches ofthe functions ei Sears une ea Car coat a ama Lue Consider praph of the fncion y= Ge the diagram), We can see that this curve tends towards both the coordinate axes. This is because there are two asymptotes: their equations are x=0 and y =0. These asymptotes are straight lines and come from the condition that x#0 and #0. To 1s 20 25 30" If we then consider the graph of the function y=—1— (as seen in the following diagram), x+l } Extending this idea even further to the function y= ** Gee the following diagram), we need to simplify the fraction before we can find the asymptotes. First, note that this function has a vertical asymptote at x= 1, since the denominator is zero. Also note that the numerator is larger than the denominator, Chapter 2: Rational functions ‘A top-heavy or improper fraction is a fraction where the degree of the polynomial in the numerator is greater than or equal to the degree of the denominator. Examples are < ey ade 3 a7 For improper fractions, the horizontal asymptote can be found in two ways. We can consider is the asymptote. Alternatively, we can split the function into smaller parts. Since the numerator of the fraction is larger than the denominator, we can assume that “+1 4 42 = x= 2. The equation can then be hi oamhen thecure y=2*! ielosta y= andso y Hence, A(x~ 1) +.B, Wecan solve this to get A written as y= 14+ £5. As fh] 0 weean see that —° 0, so y approaches |. Lastly, we consider specific values to work out the shape of the curve. When when y= 0,x=~1. Now we can sketch the eurve. Precise Skeich the curve y=2+5, showing all points of intersection withthe coordinate axes, State the equations ofthe asymptotes ae Answer when y=0,x 0001, 7<0 1,999, »>0 en Sere eo auras Rua uC ‘Sketch the curve, labelling all the important features, Include asymptotes, ‘urming points and any intersections with the axes. The examples we have looked at so far all have a linear denominator. In the next example, the denominator is a quadratic. Consider the function y = (shown in the following diagram). The first point to @DeE-3 note is that we now have two vertical asymptotes: x= 1 and v= 2, Determine the horizontal asymptote next. IF fx| + 09, then» =0 is clearly our horizontal asymptote, F For this curve, »#0 end when x=0,7'=4 You wll notice inthe diagram that theres at Teast one t ing point, To determine the turning points we can use one of two methods. ’ We could just differentiate to get ©? = — ae aa OF happens to be the midpoint between I and 2 1 3. This value also 2 ‘The other method is to start with . then rearrange the equation to give = 3x42 yx? — 3yx + Qy = 1) = 0. We can consider this as a quadratic equation in x and use Note that this second the discriminant &2 ~ 4ac. Use the condition that the discriminant is <0 to set up an. method is only inequality in y. In this way, we can find which y values are invalid for the function. applicable when the P—4ac y? — 402 1)<0 so (+4) <0 and therefore -4 0 and y < ~4. We can also see that the turning point occurs when y= ~4, er ra Tra t av+iete de extf ‘then rearrange to get (dy ~ ay}x* + (ey ~ hy)x + (fy — cx) = 0. Then, for this quadratic equation in x, using the discriminant &—d4ac <0 will tell you what values of your curve cannot have. WORKED EXAMPLE 2. The curve y= For any curve of the form itis possible to multiply through by the denominator, ——*____ js denoted as C. Determine the points of intersection with the coordinate axes, find G+ D@-3), all asymptotes and determine any turning points. Hence, sketch C. Answer 1x b]> 0,909 y=0 =1.0001,y< Oand x= -0.9999, y > 0 x= 2,9999, »-0 Consider the two curves y= and ne *> Gxt ode+®) a,b, c,d, e,f are constants, Discuss in groups what features these two curves have. Do their asymptotes differ? Do they have the same number of turning points? eT Tt aC ‘So far, we have seen improper fractions where the numerator and denominator are linear functions, Now we will consider a function in which both the numerator and denominator are quadraties. ‘One major difference is that y =F ax? + bx will have a horizontal asymptote at y Depending on the numerator, there will be 0, 1 or 2 turning points cnt) (= Ne-2) sae (x= D@=2 ee (Qx-Dor+ 1 of turning points. Answer 1 x=-lx=t 1 bls 00,9=5 ys0Sx= 1x x=03y=— oy t= 10x41 dx Qx2+x- 17 72 -10x41=0 = 4ae = 72 x=~1,0001,y> 0 and x = -0.9999, y <0 x =0.4999, »-<0 and x= 0.50001, y > 0 (ied) Dx KFT , showing all points of intersection and asymptotes and determine the number State the ve {Lasymptotes. Evaluate the horizontal asymptote. Obtain the points of intersection, Differentiate and simplify. The discriminant is greater than 0 so there are two solutions, Hence, there are two turning points. In fact, the stationary avd 22) an (4 resis ee) eae) ‘Test values either side of each vertical asymptote. points are (* ‘Notice that the curve has one turning point between x= I and . Recall that at both these values Ge ke-2) “@cnerh CPA MrT rat ‘When dealing with this type of function, differentiation can be rather time consurning, but there are ways of making it more efficient. For example, if we look back at Worked example 23 and split the funtion nto pata fraetons, weave y= 5 4 55 = 2 Qx—1P (HIP quadratic as before but in fewer steps. Pecianuaee! ‘You are encouraged to consider turning points. Sol 4 Differentiate this to give = Ving this leads to the same Find all asymptotes and points of intersection and hence sketch the curve. bxl+00, y= When x=0, When y=0,x=—-3andx=1 2x = —1.0001, <0 and x = ~0,9999, » > 0 9999, y 0 dy _-axt4 2x dx (Pf -x-2F QP 4-9-7 eeu a uC ee Te a Cur Cac Gren 1. For each of the following, determine the equations of the asymptotes of the curves. 2x43 path t3 xv -2x-3 b 3x42 xo 2x-8 ays y 5 Sketch thecurve y 6 Acurveisgivenas y a Write thisin the form y= A+ State the equations of the asympiotes € Sketch the curve, showing points of intersection with the coordinate axes, @ 7 Acureaseqution y=3=* a. White down the equations ofthe asymptotes b_ Find the x-coordinates of any stationary points. © Sketch the curve. © 8 Thecurve, ¢, is written as y=—* (= D@+3) a Show that C has no turning points. b Sketch the curve C. Show on your sketch all the points of intersection with the coordinate axes. ® 9 Thecurve y has two vertical asymptotes and one horizontal asymptote. &=DE+9 a Find the equations of these asymptotes. b Determine the number of turning points. Sketch the curve. 2.2 Oblique asymptotes In this section we will work with curves where the numerator is @ quadratic and the denominator is linear. This produces an asymptote called an oblique asymptote, which is neither horizontal nor vertical. The curve will have one vertical asymptote that is easy to identify. To find the oblique asymptote, we need to write the equation of the curve in partial fraction form first. fairs If we consider the curve es jr the denominator indicates thatthe vertical asymptote ze is x= =I, To find the oblique asymptote, we must frst consider = 4x4 B+, hence x? = (Ax + B(x + I)+C. a x41 With x=-1 and x=0 weget C=1,B=-1 andit follows that 4=1 -1+ This gives y= and the oblique asymptote is y = x — 1 gives y xt 22+ BEE si the function into the form dx+ ‘The vertical asymptote is x=‘ and the oblique asymptote is For any curve ofthe form y= c = Axe B+, ees hacke pede B, where A a EIS evericalaympioteis x ere For any curve of the form = and the oblique asymploteis y= Av+ Ryvhere A= ennai The curve, C, is defined as oblique asymptote. 41a (e+ Bx-2+C x=25C=5 and x=098 pox42 ‘Once we can find oblique asymptotes, we will be able to sketch the curve. As before, we can check 1-values ether side ofthe vertical asymptote. We can also test if curve is above or below the oblique asymptote. Look again at the function from Worked example 2.5, We are going to determine how the curve relates to the oblique asymptote, For x= 100, we can see that the y-valuc is 102.05, ‘which is just above the asymptote where y= 102. For x= —100, the y-value is -98.05, which is just below the asymptote where y= 98 Tet ae ea uaa aE eC ae TL recientes Sketch the curve y= == stating the equations of the asymptote and coordinates of the points of intersection Answer 3 State the vertical asymptote. State the value of 4 Rearrange to find the coefficients. Find Band C. State the oblique asymptote. Find the points of intersection. 1.4999, »-< Oand x= 1.5001, 7>0 ‘Examine the behaviour close to the asymptotes. 100, y, = 50.76, yy = 50.75 “Here y; is the asymptote value, and y, is the eurve value 49.26, yy = —49.25 For large positive values of x the curve is above the oblique asymptote, For large negative values of x the curveis below the oblique asymptote dete investigate what happens when you differentiate the ae ig pet yor ‘equation of the curve, How many different curves can be sketched f Foracurve of type y= this type of equation? This type of curve always has two branches, which are separated by the asymptotes and lie in two of the four regions created. or epa eerste Cone tocane = 2=4 wei sh his cane ginny tng the vertical asymptote x= 1, Nex, rewrite the function as © —4— 4 4 B+, noticing that A= 1. Then, as <= 4= («+ B)(x~ 1) + using x= 1 and x=0 gives C=~3 and a 3 Differentiating gives SY = 1 + Bee ager iggy 3, tis clear that there are no turning Soyaxe1-—2 and rearranging this as (x ~ 1} ay Setting & ing points, The oblique asymptote is y= x + 1. Therefore, the curve must appear as shown in the diagram. The points of intersection (2,0), (2,0), (0,4) reinforce thi Precis Acurve has equation y= 2 Find the equations of the asymptotes and the coordinates of any points of intersection, Hence sketch the curve. x=0Sy=~9and y= 05 x=-3, Sketch the curve. EXERCISE 28 eo 1 For each curve given, find the equations of the asymptotes. Bet 4ext3 3-31 a yode tet ef #3231 rs) - <4 Find the number of turning points for each of the following curves, was x4 Sx-4 aye b y= fe a: O° +56 Write the curve in the form y= Ax-+ B+— Acurveis given as y= given as Y= 2x1 and state the equations of the asymptotes. The ourve, Gisdenotedas y= 43. rnd te equations ofthe sympttes an, ene, kesh ax , where dis constant The equation of a curve is given as y a Given that one of the asymptotes is. = + 2, find the value of 2 b_ State the other asymptote and sketch the curve. 2x41 Acurveisgiven as y given as a a. Find the equations of the asymptotes. b Show that one of the turning points is (1,0) and determine the other turning point. Sketch the curve. axtb The curve, C, is given as y=" EO Given that one of the asymptotes is. 5 x +2 and that one of the 4 points of intersection is (0,4): a find the values of a and 6 b_ determine the number of turning points sketch C. tere ENCE ta © 5 Ancquationissivenas » Show that the curve can be written in the form y= ax-+ +=" stating the values of a, and y z= b_ Show that there are two turning points. © Sketch the curve, 2.3 Inequalities Consider the curve 22 and how w x + and how we could find x for 5 2x43 Rearrange this to get x7— 2x —3 <0 and then solve, which gives -1 0, there are two distinct intervals to consider. Using the curve er SO ea TE eC LCL Fe ona So -2-VI3-2-+ VI3 are the to intervals required WORKED EXAMPLE 2.8 X44 determine the values of x for which > 1 the equation y=——=+4 cee (2x+ 3+) Answer First sketch the curve, Identity the asympiots, Ea Always remember to ao Bet Sef pan oquon toad me ee Gar O40, cx values aay tal nrmation. 1 SMcearereil rele Sea os inert tae ieee aan -1-4W6 2, just as before, ‘The second method is one that we have met previously. Start with y= + then TOES Toa aac Cen ance ea eur Went) Determine the range of the curve Answer Method I: Syx + 4yaP— ‘Cross multiply. ax (y+ (Create a quadratic equation in x. (CaP +4(4y+5)<0 Use 6 — dac <0. (> a ) a Utey Complete the square or use the quadratic formula to Pecan conclude there are no solutions we can't have. Hence, ver Method 2: + Change into partial fractions. 9” 9Gx+4) a ferentiate, 3Gx +4) Conclude that the gradient is always positive. This leads to the same conclusion. 1 3x46 x2 4 3x+6=0 clearly has no solutions, which implies that this curve can have no vertical asymptotes, More importantly, it also tells us that the curve must have a finite range. We should also note that as |x] becomes large, tends to 0, so there is a horizontal asymptote at y=0, ‘The next type of curve we will look at is different. Consider the curve» “so 5 #0 35 30 25 20-15 10 05 “025 030 Writing the equation as yx?-+ 3yx-+ (6) ~ 1) =0 and then using 8 — dae <0 gives the ad these are the auudaiinquly 1532-4) <0-Thilads to 9 <0 or > values we cannot have. And so our rangeis 0- 0. ° ere aii 19 fae a ees ra0+y=2endya0o 20-1222 - aa IP the numerator cyazario > hhas no solutions, the ‘curve never meets the If the denominator ‘nas no solutions, then there are no vertical asymplotes. Booearis 3 @ 1 Forthecurve y= a determine the values of x such that > 3, @ 2 Forthecurve y= "5 find the range of values that y ean have Cambridge International AS & A Level Further Mathematics: Further Pure = 2x @_ 3. Given that y= —2*=4 show thatthe range of the curveis ye R, @ 4 Forthecurve y=*=2, determine the values of x that satisky y <5 xe1 @ 5 Thecurve, Gisgivenas y= x3 a Find the range of values of y that the curve cannot have b Determine the exact coordinates ofthe turning points O@ 6 Thecane y=38=2= x-2-3% a. State the value of k has a horizontal asymptote at y =k, b Show that there are no vertical asymptotes. ¢ Determine the range for y. OD 7 Theequation ofacuneis y=—— ia a Write down the equations of the asymptotes. b. Show that y ER. < De ‘mine the values of x that satisfy > 1 13x — 1-3 ___ Determine the values of x that satisty y <2 3x10 @ 8 Thecurve, Cis givenas y 2.4 Relationships between curves From a known curve, we can determine the shapes of other related curves. For example, = 3x4 2, then we can use this curve to determine the curve 1, Tia) Zp aa 2s 3236 a0 04 First, consider where the curve crosses the x-axis, These are the locations of the asymptotes. So when x= 1,x=2 the curve will be infinite and discontinuities will exist at these points. a euPa Cue at 7264 56-48-40-52-3 2432 4048 56 647. | You first saw this curve in Section 2.1. We can see that, as |x| tends to infinity, the original curve tends to infinity, and this new Function tends to zero. The original curve crosses the and So our new curve erosses the y-axisat_y = If there ate solutions to for the original curve then we have asymptotes for the new curve, xo 4x42. For y=0 there are no solutions and, hence, praxisat y the equation for f(x) Consider, instead, the curve has no discontinuities. The curve has a finite range. ext? ‘You should no fam wth bok types of asympoicare as youkne seen espe a earlier in this book. recite et P4x-2 and g(x)=x? ed 2 + and g(x) =x? + 2x+7, sketch i ™ i Given the curves f(x) Answer f) = (@+2@-1) Hence, discontinuities at x Horizontal asymptote at_y erie ea Saree Tee ua Cun manu erg Sketch the curve, Fa otiemcions (@) x2 +2x+7=0 has no real solutions, therefore there are no vertical asymptotes for our graph. Ty-1=0, yields y=0 and 3 Solve yx?+2yx-+ Use F? = dac =0. Then ~24)? + 4y Hence, the range ofthe function is 0 0) and one and y Consider the curve y=— negative branch (y <0) a Cpa Sty Notice that the green part of the curve has been reflected in the x-axis. This is true for all f(x) Now we will look at curves in the form y= f|x| curves of the form eal 4 shown below, We can see that there are two distinct branches, Consider the curve y= bad i x = :ta will have the same p-values so we can sketch the curve, This is shown in the following diagram. 2, Next, we can see that We can first note that [>| Consider the function Notice that the curve is symmetrical about the y-axis. All parts of the curve that exist for x>0 have been reflected to the other side of the y-axis. There are now four asymptotes, since the asymptotes are also reflected in the y-axis. Alllcurves that are of the form y= {|x| consist of the original curve for x2 0. This region is reflected in the y-axis to form the other half of the curve, rcianuateee Given that £09) Answer ‘Sketch the original curye. Note that the asymptotes are x43. ‘One branch should be below the Ensure that the negative branch has now been reffected in the x-axis, ‘Note that there is now another asymptote that governs the curve: eor-3. ae Cpa CII) of the cur reffo Finally, we shall fook at graphs of the form y?= f(x). Consider the curve »?= x. This is parabolic in shape. To sketch this type of graph, we use a systematic approach, Vx, Note that the First, ensure that f(x) > 0 and then sketch both y= Vx and y domain for this Function is x > 0. ‘When sketching this curve, notice that itis symmetrical about the x-axis. This is the case with all funetions of the form y? = f(x). Next consider the curve y? = 2x +3, The right-hand side is > 0 provided that x > We just need to sketch y = v2x-+3 and reflect this curve in the x-axis. 3 In this example, we should also notice that when x= -3,y=0 and y?=0, and when x=-Ly=al 2 ee a uO Te aa urea a Cur Eicon By considering y = V5 — 4x, sketch the curve y* = 5 — 4x, stating any points of intersection with the coordinate axes. Answer ‘ 5 i a For 730 weneed x< 3. State the domain of the funetion. oo when x (5 and when y Determine the points of intersection. Sketch y= V5~4y and then reflect the curve in the oeaxis ONIN ‘The term asymptote was introduced by Apollonius of Perga more than 2200 years ago, when he was working on conic sections. The term asymptote is derived from the Greek word asumptotos, which means ‘not falling together e@ Cree a tat cea ‘The curve C has equation anos eat where J is @ non-zero constant, Obtain the equation of each of the asymptotes of C: In separate diagrams, sketch C for the cases A>0 and 4<0. In both cases the coordinates of the turning points ‘must be included. Cambridge International AS & A Level Further Mathematics 9231 Paper 1 Q10 June 2008 Answer Let Ax+ B+—©, where A= 1 x+a xed So P=x(x42)+ Bx +A+C. x=nA leadsto C=#,and x=0 leadsto B=~2 So y=x- a+ and the asymptotes are y=x—4 and x= i yea ne y 2 = Simplifying this gives 4? 4 2ax-= de (+ap (4 ayP and x=~-24,y = 44 Hence, there are always two turning points, located at x= 0, » eee Sere. Vem aur eat ha OL uC EXERCISE 2D @ do not use a caleulator in this exercise. 1 Determine the number of solutions for each of the following equations. 2 — d\af -3 (2 — 2)x4 fe Ii-2 If=1 ad? 31x] +2 3. Determine the equations of the asymptotes of the curve in the following cases. 1 a yet boy= a4 Paes BD 4 Giventhar 19 = 6, sketch the curve y i) 5 Sketch the curve 3x, showing any points of intersection with the coordinate axes 6 Given that f() =, sketch the following curves. Show any asymptotes in each case. a vals) b y=flx) You are given that f(x) == +2*= a ° e x+3 a Show that f(a) has no turning points. =1 and Show also that the asymptotes are y € Sketch the curve |f(x), showing all the asymptotes and points of intersection. —L., showing all the asymptotes and points of intersection. a Panne Checklist of learning and understanding Graph types: axth a © Fortheform y= 27 the horizontal asymptote y= andthe vertical asympoteis x ® » ‘The points of intersection are (0,5) ana (2, baa Pa ( a) ( ua °) For the form y=—— “+? the horizontal asymptote is = 0 and the vertical asymptotes are x=: (ext lest) Bh fax + Mex + Z Forte form ye cralanjptota jm june the yatend aspctotea we (ext Mer +A) C7 rhepsng etna (024), (hs) an (40), fax Bort e For the form y=. Kee) where ex? + fx + g gives no real solutions, the horizontal asymptote is y = ‘Then intent ae (0.4), (-2.0) an (-He) : fractions is y= x + 2, whee Bis tobe determined «epi tenstin a (11), (5) an (-£0) For special curve types: © For yay consider all points where f(x) =0, (x) = 0 and slso f(0) =. All of these values can be used to construct the new graph, For y=[(x}| consider the original curve y= f(x) and then reflect all negative parts (y <0) in the v-axis (lub, consider the original curve for x > 0, draw this section and then reflect this in the y-axis for the complete curve, For ?=£(0), consider the domain such that f(x) 20, sketch VA(&) and then reflect this part of the curve in the y-axis Inall of these eases, make sure you consider the effect on the asymptotes. ie caren O Se yee ia aaa reat nau eee eerie) 1 Acurveis given as f(x) = Find the equations of the asymptotes of the curve Sketch the curve. Hence, or otherwise, sketch the curve - 5 State the equations ofthe verte! asymptotes 2x? + 5x1 @ 2 ecane chasematon y= 2S Find the equations of the asymptotes of C. Show that [> 2 at all points on C. Sketch C. Cambridge International AS & A Level Further Mathematics 9231 Paper 13 Q7 November 2013 2x wo2e+1 Find the coordinates of any stationary points of C. = ore © 3 Thecurve Chas equation y State the equations of the asymptotes of C. Sketch C, ating the coordinates of any intersections of C with the coordinate axes and the asymptotes. Cambridge International AS & A Level Further Mathematics 9231 Paper 11 Q10 June 2013 Cee a Ee | Bs ea oe | in Summation of series In this chapter you will learn how to: fen Acne We na ase Ug uC ee Seared ‘Where it comes from ‘What you should be able todo | Check your skills ‘AS & A Level Mathematics | Work with basic partial fractions, | 1 Split the following into partial fractions. Pore Mathematics 2&3, — | particularly those with a higher Chapter7 power inthe denominator than in | the numerator. eee | (@+D@-D ~ 3x42 ‘AS & A Level Mathematics | Have basic understanding of | 2_Evaluate: Pure Mathematics 1, arithmetic sequences, as well as | hi 20 -0.5") Chapter 6 ‘an appreciation for convergent Sar Shoe Whee SG s What is the summation of series? A series is the sum of terms in a mathematical sequence. Historically, mathematicians needed to use infinite series to approximate values such as x, and to develop polynomials to approximate other functions. Today, we can use series to work out the value of financial investments or to help musicians mix recorded tracks to achieve the sound they want. Seven) wit htc gic renown he Boo o sei atceecomnct tir cupcr encase te aceasta kong at ee end dt mai ETE Sotelo pepo 3.1 The summation formulae Zr, 7, Er? 2) kev POINT 3.1 ‘The expression for the sum of « series consisting of a constant K, added together m times, is written as ee In your AS & A Level work you will have seen a variety of sequences. We know that the a then sum of an arithmetic sequence is S,="[2a + (n ~ 1)d], For the case when a= 1, adn +1) 2 sya + (¢— 0) This results writen as 85 = Itis known as the sum of the first m natural numbers asit represents 1+2+344+ ... 4m nin i D Another way to derive this is In summation notation, thisis writen as + 2r+ 1, then write down the expression as r goes to consider the expression (r+ 1)? from | to. Carre Taegu 1>2=P42x141 2>PaPHIx241 BaP =F 42x34 n>(ntlPor42xn41 Consider the right-hand side of the equations, summed from top to bottom. Write 8 the expression ye 42D rtm summation form, This gi Next,add I to both sides toget 2427+ FP 4 eat (n4 P= DP +2 tnt ‘The let- and right-hand sides both include |), Cancelling this erm leaves nin) 2 (n+ D2 =2x Dirt 1 This simplifies to Weiecutens A « Find )'4r +1). Hence determine the value of '(4r + 1). a 7 ji Siler AR catty =n? +30 . Seesecsie Ses ye exar 43x49) ex 1843.16 7 = 4192 Since the sum has a variable upper limit, we can write anything we want as this limit, For example, we might want to determine the value of ))r, which would be m2n +1) . an ‘This result can be used to deduce the result of )’r. Carefully consider what the limits aa ‘mean: this is the sum of the first 2 terms minus the sum of the first m terms, a n+ ferent an TT CCL n+ 351 2 2" So 2n? +n = Weciecuaten! Find an expression for 143+ 5+7+--, for the first m terms, using two different approaches. Answer ‘Method 4: La Se ‘Write the sum of odd terms. Use the standard result. Simplify. P143ES 4TH ee? Method 2: 14345474 Write a sum of 2n terms and subtract all even terms, leaving n terms. ‘The second sum is twice the sum of the first natural numbers. ‘This gives the same result, The next case to look at is the sum of 14.2 +34 424 .--. We will use the same method as shown previously. Start with the expression (r+ 1)?= 1° + 37? + 3r +1, then write this for r= ton rele D=r+3x 43x14 rade PaPs3x 243x241 rapoPaP + 3x 43x941 rans (vt lPom43xn43xne1 433+ n Then ada to both sides to gt Som the ght sideso get 5) BeBe eo tee De Desay es sD tit L.Aftercameling ike terms, waSrenet this reduces to m+ 3nt + 3x41 nwt) as -, we have Ws in Ing 1 SEA D ee a es n(n +1) 2 BY Pant Int2)—3 a sEP anne (med) and it follows that) .2-= joe + DQn+ 1). ‘This is the sum of the squares of the first » natural numbers. Prenat Find an expression intermsofnfor 3°32 = 4r-+2), Answer =3x a Using an approach similar to the one in Worked example 3.2, we are going to find an expression for 12442472 + -~, for the first n terms. There are, agin, two ways ta achiewe this, Fest, rte J'Gr—2) and then expand ito a9 Prt Sa a This gives 9x dine + DQn41)- 12x yn +1 +4n Woe ean D6 Onda (61? ~ 3 - 1), = 318 +31? 43-61? —2n which simples to + ‘The alternative method is to assume we have 3n terms and write the expression as D?-Ler-wy- Yow The second sum removes 22, 57,82, Peeters from the total and the third sum removes 32, 6°, 9°, ... Working through the algebra will ead to the same result as before. The first method is simpler, but there are cases when an alternative approach is needed. ee en eur Ine ee nau en eee in hod Carl Friedrich Gauss added 1+2-+3-+ + 100 ina matter of seconds. He did this by ‘considering the pairs (1 + 100) + (2 +99) +» (50+ 51) = 5050, not 1) [oo ‘Working in groups, try to work out the sum of 1? = 27.43? — 4? + ‘This beame romaly known as JY Can you write this as one summation, or do you need a series of summations? ‘We will now consider the sum of the cubes of natural numbers. This is the sum PHES3 4 [As with the previous two examples, start with (r+ I)’ =r 44° +6? + 4r+1, then list the terms as before, from r= to r=n. palo Baty ax P+ Oxi eax ttl pars stateauDpox2eax24l radeon dx P+ Ox 44x 341 rand (nt iants4xm boxe t4xntl Following the same procedure as in Worked examples 32 and 3.3, we find ‘This is the sum of the cubes of the first natural numbers. rc rcuatee) Find anexpresion in terms ofa for r= D Answer Dere-n=Ye-Ye Write as two separate sums. = Liter 417 = inn 20+ 1) Replace with the appropriate formulae, anos (G44 E-2) ‘Note we do not need to expand all the brackets to simplify. nln + 1)@n? ~~ 2) ; saa = Lint + Dia Gn +2 Simplify. Bp + Nk 19Gn-+2) split. ee Ce ae oc} OES ‘You will meet these formulae again in (Chapter 7, when you will prove them in 2 ‘more rigorous way. Asa class, or in groups, can you derive an expression for 4 ? This is beyond the a syllabus but will reinforce your understanding of summations. Bonen © + stowinar Yar 5=%Gn-7, © 2 rrowitat Sau )=2n0 + 140-8 @®_ 3 Determine the value of 1-2+3-4+ ... + Qn—1) - Qn). @O + witnoutusings calculator, show that Yer + 240-43) = @® 5 Without using a calculator, find a simplified expression for PEP Ppa PO oe + Bn 2P + Gn 1)? - Grp? : @ 6 Motnumsetamemniorte Fre a © 7 Findthesumof the terms 124243424 04 B44 97+ 9824 992 . 41304 13) = O©® & stowstar Fars nor+2 =P * B42 sree, sn the realtor Yar+ 18+. ©@® 29 Find the sum of the cubes of the first 100 even terms, 3.2 Converging series For the series in this section, we will reach a finite sum as the number of terms tends to infinity. This will be achieved by setting up the series in an appropriate form to help us spot a recognisable pattern, How could we represent this in terms of ni? Consider the summation r+) we write down afew terms, £4 Ea ts a+ 5p te appears that there is no recognisable pattern. So, go back to the original form of the summation, and this time split the fraction into partial fractions, 1A Ay reer rel and from r=0,4 = Now we have which gives 1 = A(r-+ 1) + Br. From r=~1,B Ceri CsI esa nt a eeu eee t ) Write down a few terms, from the start and the rrel end of the series, tis always a good idea towrite down some terms To determine Which terms cancel you need to see some of the pattern. Ensure you write down terms at the start and at the end of the series z¢ Lotso he temscansel and weare in wih Sy (2-15) 1 1 ‘The cancelling of terms in this way to simplify the summation is known as the method of differences. erate 1 + Nee 2) Find, in terms of thesum of Answer uace Ae Split into partial fractions. Ar +2) + B(r+1) Find the coefficients. using= 1,4 = 1 and using Write down enough terms to see the cancellations. 3(4-4)-( -Y | Cancel terms to get the final answer. Lares For the two previous examples, the difference hetween the factors of the denominator was Lin both eases. Recall we had r(y-+ 1) and then (r+ I\r-+2) If we now consider the summation)! _.. the procedure will be the same as before Aerie) fe 1s (1 1 Split it into partial fractions fst to give LYS (1-1), then write down some term lit it into partial fa eve $3 (!-35) some terms, inal poring the Hin onto the summation JG) UEC arc Ly napgHiwe then multiply by the we ignored Cancel like terms to get 1.41 2 ntl at eee 1 r+2) 4 atl) +2) previously, the summation can be written as In this example, notice that two terms from the start and finish are included in the final answer. Previously it was only one term from each end. a Ste atat ate ett Consider the summation YY ———_1__ Fars oyar+o) changing these values. For example, what happens when a = 2? rcsnuates For the summation In groups, investigate the effect of 1 find an expression in terms of n. 4(r— +1 Answer Shen B Split into partial fractions. (= Des r=T ret 1=4¢+1)+ Br 1) Zemnern ~G-3)*(G- ae) Ag 2n Ant) a ea Gee ee a Finally, let us consider these summations as the upper limit tends to infinity. Since the powers of r are always dominant in the denominator, these summations will always tend to a limit, Pay close attention to the limit of your sum. ‘This may not be 1 +1) fr = 1 As m-soe this will tend to 1. ra Look backat the example of recientes Given that S, . find the sum of the terms as. >. (r- 1) Answer op" 4.4 Split into partial fractions. $= A(r—1) + Br A=-5,Ba5 Determine the coefficients. Tnitially ignoring the Sin front of every ‘erm, write down enough terms to spot a pattern, G-a)+(- s State S,, remembering to include the 5 we previously ignored, 100, 5,95 Write down the limit, Brent 3 (I © 2 Find interms of anexpression or SY . 1 a pay ee ames © 2 stow ina Yas canbe wttonse 5: 1 cermine the value of Y)———1_— ee Lasn@s5 1 4 Witioucusinre calle: Sil vbr oti au Vt oe ac apresiontor oo ¥ n+l Without using a calculator, given that Sy, ST Show that S,=—— @O 5» wivowr wings cateuator senha = Pe area hm Saag BOG 5 witowrsingrcaeuaor show hat a3 == Here, determine interns of, wnreantor Fob a Cees @® + wivovresmacsiuts daermine bevaneor Fy a @ sis-y Sr -sr+6 a. Find, in terms of n, an expression far S,. b Given that S= limS,, write down the value ofS, ¢ Find the range of values of n such that S— S, <0.001. WORKED PAST PAPER QUESTION ‘Verify that, forall positive values of m, 1 Testi, an 49 GEDA (WFHONFS) + A(N+ Qn + 3)Qn + 5) an+9 For the series Y’——__@#9___ ng S Lins Dn 300-4 dan 4 5) i thesum to N terms, ii the sum to infinity, aot 1 1 Qn? + Vin + 15 Qn? + In + 6) GHDRA+ GFHONFS) HD+ IOn+DAn+ 58) © (a+ Dla + In + Qn + 5) 1 1 W+20N43) WOE! 1 eer es re ETT) Boe al 4x7” x7 5x9 1 “15” W+3QNG5, fi As Neo, sum becomes is Cambridge International AS & A Level Further Mathematics: Further Pure Mathematics 1 Checklist of learning and understanding ‘Standard summations: ‘When we we the method of dferesces frst spit 3° into aria actions such r+ Nr +O) [Next list as many terms as needed to effectively cancel terms and get a Forconergens, fiat gt th summation inthe fom. fn) b+ Allowing n— 00 will ead tothe reslt of &. er Ceres Gracanica cne) ® do not use a caleulator in this exercise. li, 3x5 © 1. Find che sum of the first» terms of the series mn and deduce the sum to infinity. x Cambridge International AS & A Level Further Mathematics 9231 Paper 13 QI June 2012 @ 2 Find 2444-4 np. Hence find 12— 2* + 3*— 4? +... — @n)?, simplifying your answer. Cambridge International AS & A Level Further Mathematics 9231 Paper 13 QI June 2011 iinet apeaaeet. Qr+1)Qr+3) 6 2QN+3) x © _ 3 Usethe method of differences to show that)" 1 Deduce that » Gea Cambridge International AS & A Level Further Mathematics 9231 Paper 11 QS June 2013 Matrices 1 Cina eared CeCe Where it comes from ‘What you should be able to do ‘Check your skills AS & A Level Mathematies Find the scalar product of 2-and | 1 Find the scalar product of the Pure Mathematics 2 & 3, Chapter 9 | 3-dimensional vectors. | following a Gi+)-Gi- 4p b (i+ 4)+ 5k) + (-6 +24) What are matrices? Matrices (Singular, matrix) are rectangular arrays of numbers, variables or expressions arranged in rows and columns, They are widely used in computing processes, for instance reflection and refraction in computer graphics or computer modelling of probabilities for weather forecasting Matrices can be used to represent information such as the coordinates of an object in 3-dimensional space. You might have a basic understanding of how matrices work from your IGCSE® course. In this chapter we will focus on matrix operations, inverse matrices, special types of matrices and matrix transformations. 4.1 Matrix operations ‘The size of a matrix is defined by its rows, m, and columns, n. We refer to matrices by their order or size, mn ay, ayy a3 Fora general matrix of order mxn weave { {31 02> Each dy,» inside represents an element of the matrix. . ‘24-1 For example, the column vector ( 5 } isa 3x1 matrix and G ; 1) isa 2x3 6, ‘matrix. The size of a matrix is important when considering addition, subtraction and multiplication, A matrix that has the same number of rows and columns is called a square matrix. Addition and subtraction of matrices rormmtice: A=(3 2) ant n=($ ~f) adie ses A18=($ “2) an subtracting gives A ~ B= cs ) 103 12 Weeannot add matrices C= (! 9 3) and n= (1 2) together because there are ‘elements in the first matrix that have no corresponding elements to add to in the second matrix. Matrices can be added (or subtracted) only when the number of columns and rows is the same in both matrices. Cet a nS nee 2 ei a ura a CU To add two matrices, each element of the first matrix is added to the corresponding ‘element of the second matrix. Subtraction uses the same method, but elements are subtracted, To add or subtract two general 2x2 matrices together, we use the formula shown in Key point 4. Conn To add or ut GY Cra) Matrices may be added or subtracted only when they have the same order. The result will be a matrix of the same order. Multiplication of matrices ‘To multiply @ matrix by a sealar, multiply each element of the matrix by the scalar. Forexampleit A= (2 B)sthen ka =6(% 2), whichis (£2 (9) = (75) an =(¢£ 1¢ product of these matrices is ce be of ron) genes AB (SE Soa Cy ‘The elements of the first row of the first matrix are multiplied by the elements of the first column of the second matrix. For each row x column, sum the products. This produces new element aj; for the solution matrix Multiplying the first row of the first matrix and the ‘produces the element aj2 and so on. 12), 5 6) apa (1XS+2x7 1x6+2%8) _ (19 22 (5 2) ena r= (5 gates wre (SESE ak6t 2a) -( 3) ‘To multiply matrices, the number of columns in the first matrix must be the same as the number of rows in the second matrix. second column of the second matrix, Soir It is important to understand the process of matrix multiplication before we move on to any more examples, ocean: =(4 3) and B= (8 7), determin Given that a=(3 i) id a= (§ 4). aes 1 the solution matrix for: a A+3B b B44 c at d AB e BA Answer a A+3B= Muitiply the matrix by the scalar fist, then add the two matrices. b B-4A= ‘Multiply the matrix by the scalar first, then subtract. © as ‘The square ofa matrix isthe result of multiplying the matrix by itself, eens In general, matrix multiplication is non-commutative, as shown in Key point 4.2. For two matrices A and B, AB # BA. To multiply 3x3 matrices, we use the same method as before. However, there will be more elements in each calculation. abe\ (de \ (adtberes ** Ingencral, (* * #}x(e * *)= .* ea) Ga oes 34-1 Forexampleit A= (0-3 3) ana w=( 521), then 204, 21 4, 3 5 2 1 2 8 4-1 3.13 ar AB=(0 -23 2 1)=(-16 -1 10 2 04, 1 4, 2:12:14 Cambridge International AS & A Level Further Mathematics: Further Pure Mathematics 1 2:23 sow ses ( 6 3).so ann Proce dscuatees 03 Given that A=(_1 1 -51 find the values of AB, CA and BC. Answer 03 -2\/4 8 3 Bp m4 aml td Tiiem 1 jet me ue 51 o/\32 5) \-14 -38 -14, Remember to multiply elements in raws by elements in columns, Ll f AAPOR ft z ca=(2-1-1)( 11 1}=( 4 4- ion § i 0/ \-16 5-3 Where the row and column intersect, this shows which element is being calculated. 483\/-1 0 2) (15-11 9 pe=(621}( 2-1 -1)=(-1 313 325/\ 1-1 3, 6 -7:19, Multiply the matrices in the correct order. Ez Investigate the multiplication of non-square matrices. How is the size of the resulting matrix related to the matrices being multiplied together? ‘We have stated previously that, in general, AB 4 BA. There are a few exceptions to this rule, Any matrix in which all the elements are zero is known as the zero matrix. This matrix can 000 000 ny size and is represented by Om=| 9 9 9 wra= (22) aa n=(2 2), cenvecasesi an~ (98) at me (99) 8 AB =BA when one of the matrices is a zero matrix, 03 and BA= CeCe ace Itcan then be shown that AA?= A°A = A‘ and, in general, A"A"=A"A"= A", as shown in Key point 4.3. This result shows that matrix multiplication does not depend on ‘order when the matrices are both powers of the same base mattix, in this ease A. 2) KEY POINT 4.3 There is another special marx to ook st Consider matrix A= (" }) and multiply it by 10 6) ‘a b\ (1 0 ab a (1 \(a & ‘ab ce d)\o 1) =e a) ho iJle a= he a, ‘The matrix (6 ) is known as the identity matrix, We denote this matrix as I. 100... 0 o10 2.0 ‘The identity matrix is a square matrix of the form I= | 0 0 1... 0 ooo 21 ‘This matrix behaves in a special way. When any matrix is multiplied by Lit is unchanged. I is just like multiplying any number by 1 In general, wecan say that AT=IA = A, provided that the matrix mukiplication is allowed a enter Given matrix A= om (9) Answer G 3)*@4) o Q o @ "Solve the equations to find the elements. (1) +2G) gives = 0, hence Q) + 2(4) gives d= 1, hence Henee, B= (5 1) ‘Confirm this isthe correct mattis, Cambridge International AS & A Level Further Mathemat Let us look at the order of matrix multiplication in slightly more detail. Consider the mn) 4 9 12 250 three matrices A=(1 2 6),B=(-1 -1 1) and C=(8 1 7 ).tfwewantto 64 =1 301 4, 040, determine the result ABC, we can do this in two ways. 12 27 36 240 231 189 First, find AB=( 20 13 38 } andthen (AB)C=( 144 265. 91 17 49 72 426 422. 343, 80 77 63 240 231.189 Alternatively, caleulate BC=(-10 -2 -7},and then A(BC)=( 144 265 91 1432 7, 426 422. 343, So, in general, (AB)CD = A(BC)D = AB(CD) = (AB)(CD). We get the same result when we multiply matrices in different ways, provided the matrices stay in the same sequences, in this case ABCD. Boo 6) ans pel? © a Ab+2AB b BA ¢ AU-BA a AB? b A?-AB *()*@) » (¢)*(0 2) «62 ox(! :) © 45)*(33) 9 3) andB 510, a 2A+SB b ABP © BA? d 244347 © « 1a=() “sori a= (!-2) 2 13 0-3. 0) 7 Youare given thematrices A=(4 6 0]),B=(1 0 -1)andc 0-19, 300 1 1 s ;) eae tr where [is the identity matrix. Cae aac @ fs Thematrix A isgivenas A 20 0 -10-1 10 1 b_ Determine A*. (You do not need to prove your result) a Find A? and A’ 4.2 The inverse matrix InSetion 4 we looked at how to ad, subtract and muy matrices, Wedd not ook at division because matrices cannot be divided. Comite max equton AB= 1 inwhich A= ($ 8) to determine te mai we comide vet a ee tapas ns An augmented matrix screed by conbinig te eslimns of wo malas We ean tuned matte o hat wecax ert the fw operons org on bth the ral matinan he eoty etn Row operations are used to change the elements of matrices. There are three types of row operation: © row switching, where r,++7, © row multiplication, where 7,-+kr; © row addition, where r-+7+ kr, We will use a combination of addition and multiplication, In this example, we shall use the matrix, G Tay ‘) (On the left side ofthe dotted lines we have the elements of A; on the right side we have the elements of the identity mattia, By changing the left side to the identity matrix, the right side will become the inverse matrix of A, or A“, Use row operations on the augmented matrix, to change the elements of matrix A into the identity matrix. ourtterenton rn wnebnsathemivnio (3 § | 49). Then rot avon (8 2 £2! §).nextse nord and norte oe cana ar ¢ § ey ta 3 tiene, a= (Ff Jorwecanwrie a-t=-1(_) 3). ay » Inger fora 232 square matin A= (2 8), the inverse isgienby iN ae ‘Thin be explained at = 71 (2 6) assnownin ey poin 44. The determinant, ad—be=ae(a), | inmere atin ada! 4 Section 4.3. I. helps to determine whether or not an inverse matrix e Ce ea Oe Pea ua] ‘The inverse matrix now behaves just like a reciprocal in the sense that A'A=1 and Adct= 1 Thisissiniarto $x = .Thisss uel arnatvete dvon inmate algebra since matrices cannot be divided. Let us go back to our original question where AB =I. Multiply both sides by the inverse matrix A to give AIAB= A'T, Since A'A=T and AW'T= A“! then B= Av reciente Find the matrices Band C, where AB=T and BC Answer ATAB= AT ‘Multiply by the inverse of A. Determine the result which is B. Use matrix algebra to show the result for C. Find the value of the inverse of B. Determine that C is the matrix A?. ce, oe ‘ind the final result for c(t Fin thal en oC 2) KEY POINT 4.4 Theimerseof any 2x2 wate A= ( ) eens 7 Let us try to determine the inverse for a 3 x 3 matrix, too. aD 121; 100) Starting with the matrix A=(0 -1 2),wethenuse(0 -1 2 : 0 1 0}asour pall 2317001, augmented matrix. Again, our purpose will be to convert the left side of the dotted line so that it becomes the identity matrix and the right side will become the inverse. Again, we will be using row operations to reduce elements to 0) 12 Start with the row operation ry ry to get (0-1 0-1 a get 1 00) Then apply ryory—7 toget{O -1 2: 0 10 0 0-3: 2-11 Notice that there is now a lower triangle of zeros. This is known as row echelon form, 1 Sy 1 20 Then apply rr, +2rp togivethematrix(Q -1 2; 0 10 0 0-3) 2-11 $i Ba is Follow this with r+3r, + Sry and ry-+3r)+2r5 toget(0 -3 0; -4 12 0 0-3 : -2-11 Lastly, apply r)-> Lem —try and +t ry to get the augmented matrix mei OW eee a 5 3 aed o10 Bee 34 4 oe ats 1 21 2 ee a oe Sree a) 3 a The right-hand side isthe inverse matrix, A'=| 4-1 2], 37303 Aa 33 fi Weecan also write thisas A“=4( 4 -1 -2 ). te is generally better to take out the 24-1 fraction as this makes calculations simpler. ‘The left side is now in reduced row echelon form with ones in the leading diagonal and zeros everywhere else, Prisca 41-1 Find the inverse matrix for B=(2 0 1 3-2 4, Answer Using the six operations below: ry94— 3r, 3 2n=n ronan nal rr Mn + 3h nontn 56 0 (0: 16 -16 z t take more t eadsto:{ 0 -14 0 : 10 ~38 12 - eae aps 0 0-14; 8-2 4, ea rie esi Cure 2 Vea aunt ea uC ‘Once in this form, turn the elements on. 1 Ao0 7 the left into Is by dividing by 56. —14 z ‘and “14 respectively for each row. Then: | 1 0 id 7 ‘The left side is now in reduced row TT echelon form, the identity matrix. ‘This then gives the inverse matrix on the right side. Factoroutthe + and write the elements as integers. a 5 ae. boas ve bier ais consterciemas €=(2 4 7) ayostumie se option naa a 21 4 ret, hemaistsones (04-8) Westpoint nmi 03 identity matrix. Any xn matrix with a repeated row cannot have an inverse, since one row combined with another will always create a row of zeros. This means that the matrix C has no inverse. Any matrix with no inverse is known as a singular matrix. Conversely, any ‘matrix that does have an inverse is known as a non-singular matrix, recente Remember that there are six elements in a 33 matrix that must be converted to 0 to determine the inverse. “This means that, at most, we will need 10 perform six row operations Determine which of the following matrices are singular 12-4) 12 aa=(61 2 b B=(41 2 37 -86, 06 Answer ° ao 8 rents an nana tao 0 5 { 2 = Then ry+ 2s togive( 0 -11 26 = 0-11 26, The repeated row shows us that there is no inverse. This means A is singular, non-zero, Perform row operations until you can sec if rows 2 and 3 are related or not. 1 3 b rrory—4n, and ry-+7rs+ 672 lead to ( -.) Ifa matrix can be written in the form 0 0-39 ah and so this matrix is non-singular 0 de) thenit must be invertible, 00 f) fis seiteetiils provided all values present are te Z © ror 2h, and rysry— 37; leadto[0 —3 ~ 0 -6 -12, " a? Then ry+575 togive (0 -3 -6 ew: So Cis also singular, ce a rae tin What would happen if you performed column operations instead of row operations? Would the same results be observed? Can column operations work on augmented matrices? conatetveratces A=(! 2) and n= ($ 6 eee 4(3 i) and B- rvs consider te mats AB which (12 22), en ime 11 50-22 (as) nto 3) By considering BA! We first determine the inverses to be A~! we get Lf 50 ~22 eect g\ 43 19 In general, (AB) = BA The order in which the matrices occur in the calculation is important, Note that (ABy! 4 ABT wn cms A= (F 2am ({ 2) ana c= (9 “ett relationship between the matrices A~!, Bland C~1, and the matrices (CAB)~! and (BAC) Lz0 Thosoneprini ono any xn ma So ahen A= (221) and $32 1-1 2 s-(0 0 ‘dom can determine the result of (AB)! as B-!A~!. However, to save time, it 1 42, is best to find AB first as we only need to find one inverse. 12-10 3 3 0.3 )and the product, BIA, 4-4 2 Inthisexample, A= {3 2 -1), W6 6 2, 0 50, 1 [0 30 40" works outtobe {30 30 0 1s 10-5, ‘Cambridge International AS & A Level Further Mathematics: Further Pure Mathematics 1 1-1 8 Working out AB gives the result (1 4 8 5 5 10, Following the method shown earlier, with six row operations we will arrive at 1 {0-90 40 (aBy'=7G{ 30 300 15 10 -5, ‘The advantage of using AB followed by (AB)! is that this requires, at most, only six row operations to find the inverse matrix, The alternative method can take up to 12 row ‘operations to complete. Pecans determine the result of C, where ABC 101 Given that A=(0 11) and B= 110, Answer ATABC = A“ > BC =A“! ‘Use matrix algebra to determine the matrix C BBC =BA' > C=B"'A7 Use the result (AB)! = BA, Hence, C = (AB) 1o01\/-11 0 ap=(011}{ 10 1 Determine the matrix AB. 110/\ 00-1 oY ff =(10 0 ol. Using the following row operations: ‘Apply the following row operations to turn the nontn left side of the augmented matrix into reduced nRoncn row echelon form. 100g O19 gives { 0 20 1 gee Factor out values to change the left side into I 002 114 Hence, (ABY State the result oor Faycomsier emaic A (12 6). tenets nee ome 131 semay soi that he top fow shoul ey be at the btom.Thiacan eid by dritlag rom oor: 108 ss tratiouauznmntancne (129° 81 8) 13t. 001 Apply the following row operations, ry 6 r3,%99f2 — ri, ryar + 32+ 2g and rpg + 1 00;2 to got the augmented matrix (0 -1 0: 11 o o1: 10 CoCo Notice that switching the two rows saved two row operations since the bottom row needs to bein 0 0 1 form anyway. In this example, we were also a little adventurous. The row operation ri-rrj + 3rz + 2ry saved a little time, too. Since the numbers in the example were very simple, the row operations were more straightforward, EXERCISE 4B ® + ciara a=(3 !) ans e=(2 1 dnacsash sa aCH=1 @_ 2 & Determine the value of k such that a=(5 i) has no inverse. b Given that k=8, determine B where BA?= I. 13-2 3. Find the inverse of the matrix A=(0 4 1}. nei “(3 62) 64) “(bu) a O® 5 Gircoa a=(} 4) fna the matrix A and, hence, show that A(A — SI) = 21, where Tis the identity nati Romito sow ha nme mai °'=-1( 4) 6 Determine if the following matrices are singular or non-singular. ne 1 at a(2 87 b (-1 20 110 11 3 66, 102 @ 7 Youare given the matrix A=[2 1 1 O11 a Find A*and A’ b Find the value of k such that A?—kA?+2A 41 =0, © Hence, determine A“! Lo? O10 @ & Giventnat A=( 0 2 1)andinat B= (—1 0 2), determine C, where ACI -1 10, 200, |. find Bsuch that AB @ 9° a= Cambridge International AS &A Level Further Mathematics: Further Pure Mathematics 1 4.3 Determinants In Section 4.2 we looked at the inverse of a matrix. This was briefly linked to the determinant of the matrix. conitertnenatx a= (5 2) L 2) ae then ‘a B L (a try to find its inverse. Using A = (4 7) we know that A“ to 7 (: i) ad (2 e) DB we xtnlf OF ipo ets 4) We can see that the determinant, det(A) = and the matrix A has no inverse. should lead to an inverse matrix. To confirm, we look at det(B) = 2x7 ~ (-! of course, non-zero, which confirms that B™! exists. 5x 6) =44, This determinant will always This can also be confirmed with a row ‘Any matrix that is not ofthe form G x2) swe, bare both non-zero, will havea determinant that is non-zero and it will also have an inverse matrix, \s shown in Key point 4.5 Another way of writing he determinantis [2 fl =ad—be, P) Key POINT 4.5 We have not yet Jooked at the determinant for a 3x3 matrix, so consider the 1 63 163 matrix A=(1 4 8), The determinantis| 1 4 8 | and this can be solved 5 20 4, 5 204 by multiplying each top clement by a corresponding 2x 2 determinant, The ‘overall determinant is made of three smaller determinants called minors such that =u 8) ia|t 8 1 4) which works ou e ir aeca)=if§| oft &] +3] which rks ote Hees ‘These signs when multiplied by their minors are called cofactors, and they are determined by considering (—1)"*" for each element. Lt Perens B= (2.7 4) edna d= 4d This works out to be 33. ene cate Find the determinants of the following matrices, 11 0 2045 12 3 a (210 (0 13 (02 4 312 1-13 00-1 Anoner 110 | a/210f-[1 I-B s rig) aa ea jo tl Ee ¥ [4 2 4|_,Jo 4) Jo 2 e{o2 4 -2 _f]+3]8 2=-2 110 comatose (1 2 8) sedans FY] [hf 214 vio veins teoetin manar wesa (#31) niedaeina vos ou ae ve fF A]-[P 3] =2.somisow epeton isn aff on he atria Aid £f jo 1 Next, if we use rs+rs—2r; toget{O 2 1 then | 1 d-( 4 ‘80 still no change. 0 =-1 4, a 21 Bs 18, or 9x2, res 10 9) ~ Jo 9] So this last operation doubles the value of the determinant. 110 Next, use 73+2r3 +72 to get ( 2 2) tn 351 One last operation, using 737, +12 gives us (: 2 :) ‘The determinant of this matrix 009, 0 2| 0 0] = Shor 18x3, oreven 9x2x3, So it appears that any row operation that multiplies the row being changed by a factor & scales the value of the determinant by this same factor k. eee AC Eee Pua ana Take, for example, (4) whose determinant isa Bo, snd apply the ro a b 7 ad — kab ~ be + kab = ad ~ be, so there is no change in the determinant, But if we apply rp-skra — ri, then our matrix is Ge oe s) ‘The determinant works out to be akd ~ ab ~ bke + ab = k{ad— be), which is & times bigger than before. Investigate the effect on the value of the determinant of switching two rows in a matrix. a3 1 How can we make this useful? Consider the matrix (0 3 5 }. Its determinant is the 00 4, product of the elements in the leading diagonal, which is 23 x (4) So if'@ matrix can be reduced to row echelon form, then the calculation of a determinant is straightforward. ruettemari 82 (3 43) uss peti ty =rarne= ipl ‘ 1, so tl tt iet(B) Eurxt 6. cowsine| 3 1 “bea 4) O-B 4] eto soeeers 1-1 3] 5 : Wrist) Find, using row operations, the size of the determinants of the following matrices. 208 3-1 -3 a(i-14 b(o 41 3 1 2; 1-1 2 Answer a Operations F421 ~ 3rj,r-42— ry and ry-+r3 +r PRE Valves from 2ry and 2rs make the Sf ox determinant 4 times bigger, We need to scale reduce the matrix to (: :) this down by a factor of 2 x 2 to find the value 0 0 -8 of the determinant. (The third operation does 2x (-2)x (8 not affect the size) So the determinant is —~"——~ b Operations ry-3r5—r, and ry a a matrixto(0 4 1 0 0 19, 3X4x19 _ 3p 3x2 So the determinant is ic a? = (2 4), we ean fin ‘ir determinants to Ginn womatiss A= (2 8) and n=(_3 4) ncn fn cite (4.3) st deerina vos 242. Now 242 also happens to be 11 x 22. = 22, Ifwe then consider AB out to be 52+ 190 Ba (ff) andits dstrminnat is sso 2, Hao 2 100 Using another example with A=(0 1 3} and B=(1 1 1), working out their 1 -1 0, 302 determinants, we get det(A)=8 and det(B) = ~3. 1 21\/2 10 7-1 4 Then AB=(0 1 3](1 -1 1}=(10 -1 7), and its determinant works out to be 1-1 o/\3 0 2, 12-1 baffle tea) 9 “to areee ip ecaoW Lica aN iifatis is) Also,BA=(1 -1 1](0 1 3}=(2 0 —2), andits determinant is also ~24. 3 1-10/ \s4 3, So, for determinants of size n x n, det(AB) = det(BA) = det(A}det( 24. Discuss in groups, with the aid of your teacher, the process for finding the 1 2-14 decrminantotthematixa=[2 3 © 1). Does the row reduston method O23 speed up the process now? Boers 1 Given that A (; :) and B ( : 3) conte that det(AB) = det(A) x der(B) and 15 63, det(BA) = det(A) x det(B) n 4 © 2 Tuematsicdissuch har A= (F" ,). Find the values of such that As singular @O 3 showthat B= (: ni) can always be inverted for all values of x. eran eS Raa ere | 4 State which of the following matrices are invertible. 5. Find the determinant of each of the following matrices. 1 8 6 c (0-8 4 0 0 12, efi 3 45 122 a (3 14 b (3 0-3 3-5 3, 4 3 fon rrcr) — ary changes the determinant by a factor of c se 2 14 (3) > (444) © (44) ads © © Youare given the matrix A=(b eh Show that the row oper: x ay @ 7 Giventhat | 32 a “0 |-=2, find the value ofthe constant a “ae 123 1 32 8 Giventnat A=(0 14) and B=(2 3 0), find the determinant ofthe matrix AB 120 1-10 4.4 Matrix transformations Matrices can be used to represent certain transformations, For example, if we consider the ale A(1, 1), BU, 3), CUS, 3) and D(S, 1), we ean put these coordinates into a matrix to get ase cl ‘as a position vector. If we multiply this matrix by a 2 x 2 matrix to produce a transformation, we need to multiply in 01 31)7\1331 multiplying by the identity matrix. tacos Forums (!°)(} 1 $ $)=(1 1 $$). meio shang sf 1 o\(1 155 yi iatrices gives (5 5 § 9) sensu sov dso This te shane bset on he sansa each vertex rom the axis The origina shapes ine andthe tanfomed imag sin ue ‘Next, consider the rectangle with vertices (1, 1), (1, 3), (4, 3) and (4, 1). Avamaixthicenbewsitenss(1 4 4), urvematipyiteytemaric (2 9). weas (? 2 § §) andeach «values dub, direction: the distance relative to the y-axis has This isa stretch of scale factor 2in the x been doubled. Combining the previous two examples, we can see that the matrix ( ) has the effect of cnoshramentvale tor? Hee necin sete nati (2 2) spot 1331 2662 (15 5 1) toproduce thematic (3 $$ §). This sime the entgementis measured fom the origin, so we can say the origin isthe centre of the enlargement, In each of these three examples we can see an invariant line or point. In the first example, any point on the line y= 0 will not change its y value. Itis, therefore, invariant In the second example, any point on the line x =0 will not change its x value. It is also, therefore, invariant. In the third example, both x=0 and y= 0 are invariant lines. The point where they meet, in this case the origin, is known as an invariant point. The matrices for stretches are shown in Key point 4.6. eet uu Ce Pea EL OTET ine Fora etch nthe xrstion of sl ato the mats represented by (6) For astetch inthe yietion of se factor thematinsrepresnedby (5) Foran enlargement tine the point (0,0) fs fo thematic resented by (2) [Any point that is unchanged by a transformation is known as an invariant point. If chat point is not ‘the origin, then it must lie on an invariant line ‘The origin is always invariant for matrix transformations 1 28 2 2 | | toit. The 66 7 Now consider the matrix ( 5) and apply the transformation routs (1 1 $$) wetmenscalctr ap, bhitinetiss ding fico: What 1 ° 1 } Applying an inverse 2 matrix will revert any transformation to its original state since AIA, Inall of the previous cases the area of the shape is increased by a factor that is equal to the size of the determinant that is transforming the shape. Take, for example, the matrix ve ( ’) This stretches by factors 3 and 2, $0 the area of the new shape is six times as, large since jdet(A) Gon (4,5). What do you notice about the area of the image of the triangle? How does this relate to the determinant of the matrix being applied? ‘As well as stretching and enlarging, we can also consider rotations and reflections as transformations. Consider the triangle represented by the points (1, 1), (1,4), (4, 1). As always, we write Se CC nates in matrix form (| } 4 ace aurea (4-4 A) teenth rotetnin pai ti CoG sy Ca4 the effect of this matrix is to change the sign of each x value but not to change the y values this gives é a 4) which represents a reflection in If we apply the matrix (0 a the x-axis, So these two matrices are responsible for reflections in axes. Le 7 te temsoatin ("7 2) andi eiesmaton (3°) ria ingle (1 4 )> When we apply both of these transformations, in either order, to the eenaeene (5 (4 9)(044)=(1 29) ‘This effect is a rotation about the origin by 180°. Aurea Uae TCC oee Consider the pont (x0) yng on the wax. Then apply thematrix (" f) so shat » a bY (x) _ (0080) here @ represents an kw jation about the origin; (2.2) (3) = (S288). wnste presents an antilock rotation abot te oi — hence, a=cos@ and ¢= sind. Next, consider the fact that the magnitude of this position vector does not change, so we know that the determinant must be 1. Then we have another equation ad ~ be = 1, or deos@ ~ bsind = 1 Making use of cos*@ + sin?@= 1 we can find solutions to the equation: b = ~sin@ and d = cos 6. ‘cos —siné) sind cos8, ‘Temas fran anticockwise oan bow te ois (' 0-1 So fora 90° rotation anticlockwise about the origin, the corresponding matrixis (9 ~}), a and the matrix | J would result in a rotation of 135° clockwise about the origin. WORKED EXAMPLE 4.10 Describe the effect of the following matrices on the triangle given by (1, 1),(3, 1) and (1,3). Illustrate your findings. Answer Coan) un a2) 1-6) ay ee artes h ee Pen eur nee end ues arc Ue cach point is translated parallel to the x-axis since the new x-coordinates are based on x+y. This effect is called a shear. The distance through which points are displaced depends on their distance from the line x y 6 6» ey a ‘A shape being sheared in the x-direction. Consider the rectangle (1,0), (3,0), (3, 4), (1,4). If we apply the same matrix to these points, we get a different shear. This time only the top points move since the two bottom vertices lie on an invariant line, y =0. 69 39 0) Another shear in the x-direction where the bottom vertices lie on the invariant line, Amacathe‘om (1°) sold gestern feta thy drston nsed Apply this matrix to the rectangle with vertices (0, 0), (0,3), (2,3) and (2,0) ruinsver (}2)(98 We can see that the line 1 So any matrix of the form (i ) will produce a shear in the y-direction. Apply the matrix (3 ‘) to sets of points and investigate with different values of k What happens in the cases where O- sivingea seiceueerae teas ictal Divide the first equation by the second equation to get pein _t then solving gives 2m + 3m? =1+2m or 113 Gi b rerthiasoman, (2 3)(1 4 $)=($ 4 2), rimneet. 8c tina, i 7, 10) © The inverse matrix of Tis T™ D Since the determinant of T is 1 there is no difference in the area of the triangles. en SOS ee Rg ee a EU oes Checklist of learning and understanding © 2 =(4)= (es 234) © (226 1)- (G23 73) (© For square matrices AX AX AX... XAS ° 0 ° (© The identity matrix is square matrix of the form I= and it has the property such that Ax A= or AIXA=I. Inverse matrices: is Ces ee ror 2 mane A=(22) ae tt ( $9) For nxn matrices we ean use row operations on an augmented matrix of the form abe i 100 defi dio ghi: 001, (© For any two square matrices A and B, (AB)? = B1A~! ‘© A matrix without an inverse is known as singular. (© A matrix with an inverse is non-singular Determinants: ab ¢ meseminnote 43 wan a=(#¢ 7) setae ai ‘When the determinant of a matrix is O, the matrix will be singular ant changes by factor k when row operations of the form ovr, + my are used ‘The value of the determinant is also the factor increase of the area, or volume, when the matrix is used as e transformation, For two matrices A and B, det(AB) = dei(BA) = det(A) x det ‘Transformations: © The following transformations are for 2x2 matrices ‘Stretch by 2 scale factor of factor kin the xeirection ‘Stretch by a scale factor of factor kin the y-direction Enlargement with centre of enlargement the origin by a seale factor of factor k Reflection in the axis Reflection in the -axis Reflection ia the ine Rotation about the origin by @ in the anticlockwise direction ‘The following transformations are for 3 x3 matric Rotation about the x-axis by angle @ in the anticlockwise direction 0 cose —sing 0 sind cos9, Rotation about the y-axis by angle @ in the (° sos 0 i) anticlockwise direction sind 0 cosa, Rotation about the z-axis by angle é in the anticlockwise direction e088 sind 0 sind coro 0 ‘Enlargement with cenire of enlargement the ? origin by a scale factor of factor k 0 008 Invariant lines: © For 2-dimensional cases, use (4 5) ( * {0 determine two equations ofthe form 2 cd) mt, at + bt = Tet dnt = mT. Divide to get 2+ edn invariant line() ofthe transformation in the form y= mx then solve for value(s) of m to find the Chapter 4: Matrices 1 Ere ea ne) s cienit a=(58) ma m= (“t 2) a find Csuch that BC=A+A* b determine D, where ADB = 12 2 Youges natn A=(2 3 73) oars a If the matrix is singular, find the value of the constant a b If a=4,find A“ 3. Th Aisgivenas a= (3 4 we matrix A isgiven as A= (5 a Find the invariant lines for this matrix. b The matrix A is applied to the vertices of the triangle PQR, and the resulting image has vertices at the points (11, 12), (19, 15) and (40, 43). Find the coordinates of the vertices of the original triangle. eit ane OSe eau Crea aC CL Poeciecaracd Where it comes from | What you should be able to do | Check your skills AS&A Level ‘Work with parametric curves. |1 a Given that x=2cos@ and Moticmats Pare Mathematis 243, — | Chapters find y= (x). Ir 1 rd and relating x and y. [2 Evaluate the following. Mathematics Pure Mathematics 2 & 3, Chapters 5 & 8 | What are polar coordinates? In AS & A Level Mathematics you met Cartesian equations and parametric equations. ‘Now you will earn about another type of 2-dimensional coordinate system, polar coordinates In this systom, a point on a plane is identified by an angle anda distance ether than two linear dimensions. Polar curves are described by the distance from the origin, , and the angle formed with the postive x-axis, Polar coordinates ate used for applications involving cireular geometry or radial movement, where rectangular (Cartesian) coordinates would be awkward to use, Engineers ‘use polar coordinates in the design and manufacture of gears. Polar coordinates also help scientists 10 understand electric and magnetic fields. Polar curves can be used to create some beautiful graphs, including cardioids and rose petals. 5.1 The polar system Rather than using Cartesian coordinates (x, y) to describe the position of a point we use polar coordinates (, 0). We define an origin O called the pole, and a line ina fixed direction, conventionally the x-axis, called the initial line ‘The point, P, shown on the graph has polar coordinates (r 6). rs defined as the distance of P from the pole. The angle OP makes with the inital line is 8. Note that if @ is measured anticlockwise from the initial line the convention is for 0 to be positive. If @ is measured clockwise from the initial line the convention is for @ to be negative ‘The initial line, which is the x-axis when using Cartesian coordinates, is where @ = 0. The y-axis for Cartesian coordinates is, where @ 7 for polar coodinates. Instead of writing functions in the form y = f(x), we work with functions in the form r= (0) ol os 6 28 32 4 eager Using this right-angled triangle, and some trigonometry, we can P deduce that x= rcos@ and y=rsin8, We ean combine these using the identity (or use Pythagoras on the right-angled triangle) to deduce B ” 27+)? =77. These equations are useful when converting between Ba polar and Cartesian coordinates. Note that this equation does not imply that the graph is a circle. I:merely describes a geometrical relationship between the and y values at every point on the curve, The correct way to write this is This is shown in Key point 5.1. HO)c0s0, y = i(0)sind, Remember that r= (0). ‘We would only have a circle in the case where f(@) is a constant, Olete Foral polar curves »=reosd, y=rsind and 324)? =? => r3 0. The function is given inthe forma 7= (0). ‘Changing from polar to Cartesian form is quite st For example, if r=2 then using 22+)? Pwo get 44 )?=4,Thisis a circle with radius 2 and centre (0, 0). Consider a different example: start with y sind and mulkiply both sides by 7. Using ?=22+y? and y=rsind we get x24)? =, or 24 ?—y 2 By completing the square on the »terms we find 2? + (- ’) 1 Thstahe sci cme (12) ana 1 We vtbokattiseane sein Moker esate Convert the following polar curves into Cartesian form, a r=secd b Peos20=4 Answer L a ret e088 reosd= 1 x=l b P(cos?0 sin?) = 4 Changing from Cartesian to polar form is also relatively straightforward. For example, if y= 1 then rsin@= 1, the polareurveis r= 1 or r= cosecd Sn erie a Sue aca ar e Ca LEU ee 8 reciente! Find the polar form for the following equations. a saxtey? Answer a rsint@ = P(Psin* 9) = n'a 1)=0 int@=1 = cosec'@ sin®@ b Peos?@ Peost@ — r'sin® 0cos? @ Peos? @— rsin?® A(1 ~ sin? 4) = r4sin* (cost @- Psin?@) = cost = Pin 2 6088 © sin? oe P for the Use » right side, sin@ for the leit side, and 0 is not our curve, so divide by r? (or factorise out). Find the equation of the curve. Use x=rcos@ fortheleitsideand y= rsin@ forthe right side, Use cos?@= 1 ~sin@ to simplify the expression, Factorise out the ? term. State the equation of the curve. ‘Now we shall look at sketching polar curves. The simplest type is the curve r = a which isa circle, The distance of each point from the origin is a If we look at the curve changes so the curve formed is a spiral Note that the curve tends to the initial line as 0-0 a6 , this is slightly different, As @ changes, the distance also Te ee TCTs = = on 2 = 2 137 aia [a7 om (One very useful tool when sketching polar curves is a table of values for (r, 6). From the table we can see that the sketch matches the values. ese Sketch the curve r9= |, for 0 <0 < 2x, noting any special eatures of the curve. oat 01s) 40, therefore consider 0-+0. For small angles sind = @ so Therefore, rsin@ x 1> ya 1 Hence, there is an asymptote at y= 1 eee 0 NGO] ow v5 0% toy ms a) Ls a as aS) Since polar curves are defined with r > 0, we need to consider some restrictions for certain curves. For example, if we attempted to sketch = In@, the domain for @ would have to exclude <1. For the domain 1 < 0 < 2x we would have another spiral curve, The red dashed line represents the section of the curve that we can’t sketch. The solid blue line is for > | nt a unin C Cee eau anit eam CLE UC eg 20-175 150 -1.35/-1100-075 050-0 Consider the curve r = c0s0. This curve can be completed over an interval of x. But if we try touse 0 <6 < x, this will produce negative values for r for the interval «| this in the following diagram : | You can see eee oR Ud So in order to sketch 239 ovsay wera sete interval [8 sine this interval is non-negative for cos@. If you are not convinced that this is a circle, start with cos and then multiply both sides by r to get r° = rcosd, or 32 +)? eae sin@ using a table of values. State the domain for @ that ensures > 0. This simplifies to (x Tmstacicrwincrte (1,0) anda Sketch the curve Answer a 07 06 0s 4 03-02 -01 0 01 02 03 04 OS “o1 Valid for 0-<6< x. : Earlier we converted ; cel One yong an t's cures r= an ind are 5 radians apart, erie Seu eI ere Ve a aurea Ug a nu oe In Cartesian form, this is @ translation but in polar form it isa rotation, We can see from the previous graphs that rotating r= cos@ by 3 anticlockwise about the pole gives the curve r= sind, Let us now consider the eurve r= I + cos@, At first this looks like it could be a larger circle, perhaps circle of radius 1 To be sure, it is best to construct a table of values, as shown. olelz/#|/>|#|=]2 a,2|¢[*|el2 pan [1 fo2s 0 [ozs] a [am From this table, itis possible to construct the curve, noting that there are no negative values of r. The smallest value r can take is 0, and at this point @ cusp is ercated. This curve A cusp isa point where is known as a cardioid, or heart-shaped curve, eel ri curve meet, such that the two tangents of the curve are equal ere eee ely Notice that this curve, just like the function y = cosx, has a line of symmetry at @=0. ‘This is generally the case with polar curves that contain factors of cos@. 1 + cos@, However, it is in. is, If the angle is The curve r= 1+ sind is the same shape as the curve symmetrical about the line @=* in the same way as the curve y= 2 26 rather than 6 then the relationship between the sin and cos curves still holds, but now or rather 0= 2 # We can see in the following diagram the relationship between the curves r= cos20 and in 26, Note that there are no negative regions. the angle between them is 20= Rerrctscua te Sketch the curve r Answer olt lel]: 21/212 = 6i]a{[3s{3s]a]e]* of 1 [om] o [o fowl a fo fo i termine allowed, en a EOE Ve aura te LUC ee The curve has three petals, or leaves. U1 02 03 04 05 06 07 os OD 10, Investigate curves of the form r= cosk and r= sink@ for odd and even values of k, ‘You must consider the points of intersection of polar curves with the initial line and show them on your graph. The initial line will not always be when @ = 0, as you have seen with. both r 1 ang r= Ino. 6 then use the value @= 2n, provided that If your function does not permit the use of @ your domain allows this value. @® Do not use a calculator in this exercise. 1. Find the Cartesian form for the polar equation r= sin@. 2. Find the coordinates of all the points where the curve r= cos +2 meets the coordinates axes. sind, for 0<@< 2x, Sketch C. 3 Acurve, C, has polar equation r 0, Find the polar equivalent. @_ 4 ACartesian equation is given as x7 — y+ oleate Tarr 2sind for the interval [0, 2x]. Sketch also the inner loop and state its domain, 5 Sketch the curve r 6 Sketch the curve r= sec?@ for the interval 0 < 0 < 2 320. Show that the Cartesian form is (x? + y © 7 Apolarcurveis given as r 8 Sketch the curve r 056, for S0. So we will assume that the area OPQisa sector with area pr. ‘Once we have set a limit, we can use d9 instead of 6. Adding these small areas over an interval a < @ < f, we can write Aft 6. aif ‘This is the area inside a polar curve over the specified interval. For example, let us look at the area inside the polar curve r= 1+ cos over 0< Oz, Start with A if "P40, which leads to $f. +eoroyea 0 2 Expanding leads to fo +2cos0 + cos*#)d0 and simplifying leads to 1 Jc0s26. +5 ) [(gesco0s eos) iow meat se So A= [22 4 eect Find the area enclosed by the curve r= cos29 over the interval (0, 2x]. cos? 2640 even Cie Ree ecm cua a Cress i e+ isin « Use reduced limits. Evaluate the total area. U5 Lena ® wt we evaluate 1a 1sin4a) ~ we get ®, which is twice as large as the true answer. This is Trwe evaluate 3}0 + ;sin4d] " we get 5, which i twice as large ast because the area of the curve for the regions with <0 has also been included. ENA Polar curves have been known for about 2200 years although they have not been used formally as ‘coordinate system for allthis time. Archimedes described his Archimedean spiral as a function whose radius depends on the angle. However, it was not until the 17th century that mathematicians such as Cavalieri, Pascal, Newton and Bernoulli started to write functions in polar form and use them to determine results such as the area inside an Archimedean spiral, Always use the symmetry of curves to help you work out the area of a potar curve, Finding the area beiween polar curves is similar to finding the area between Cartesian curves. Consider two curves, 7, and ras shown in the diagram. * ‘The aoa between them canbe found using 4 = [63 — rao (Note that this is not the same as af = 71)?d6: that is a very different integral} fe For example, consider the two curves r= sin@ and = 2sin@ over the interval = Note that 2sind is the interval F< 6.< 5. Not the bigger curve, so square and subtract the functions, Integrate fa sin®9 — sin?@)d6, which simplifies to . L 3* This integrates to give 4 if. 3x 3h — cos 20) dé. eas cere Tar enue) Find the area contained between the curves r=@ and r Answer , © - aaa 787 If we want to find the area between r= cos@ and r= sin#, we can see that the curves meet twice. However, these two points are Cartesian points of intersection. The only true point of intersection in polar coordinates is @ 2). As the two curves have different @ values when ), we must take care when integrating. 1 02 03 04 05 06 07 08 09 Pay close attention To find the area between them we must consider the limits of both each curves separately. Seams if a secret So A ‘sin? 040+ aI cos” @d@. We cannot combine these two integrals integral. If they differ, 5 P you emt combine because the limits are different. Use the double angle formula for cos28, to give fis nisgaieeataean| if if Sounds som, 1 0320)40+ 2,51 + cos20) 40. 1fi a —cos2mdo+ 20 + cos2mps, ea ies : neat Oe lee fl 3 - ina honeedon Integrate to give A +4o+ Sinz Evaluate this to give A= 7 — 1 os th oben ‘The Cartesian points of intersection may not be actual points of intersection lo F 4 Because ofthe geometry of the 1wo curves, we can obtain the same area by considering iff 2x} )'sin? ede, 2x5), sin?oue, ee a AEN Rusa nurses Peat Find the area enclosed between the curves r= sin@ and r= 1 ~sin6. Answer Make a sketch of the curves to identi posing — ‘the enclosed area. rl sing— Aw 075 05) 025 035 030 0.25 os “045 =1.00 12s 1.30 Recognise that the two areas on each side of the line @ =5 are equal, so you ‘only need to consider one of them, 0s 010 ois 020 Ds D3 4s a4 O4s O80 O55 DMO x [is ea0+ [7a ~sinay? ea) Use different limits for each part of the ot 3 area. = fo ~ cos 26)d0+ fe Use the double angle formula for 0320 twice Integrate the parts separately. od sinza]® + 22+ a4 2 Evaluate the area. Cea este urd Polar curves that are looped usually contain sin(1(@)) or cos({(@)). Looped regions have a maximum distance from the origin. To find this maximum distance we can look very carefully at the curve: at the point P the curve is at its greatest distance, but on either side of this point the curve gets closer to the origin, DDS 01 015 02 03503 055 04 OAS OS O55 06 065 07 OTS 080K 09 “bos Sokmustbe testa 60 at Thats rents his pon Alo at isp he line OP is perpendicular to the tangent to the curve. Consider the curve r= esin6, which is a closed loop over the interval (0, x. 600-5 o=0 S6-52-48-44-40-36-52-28-26-20-16-12-08-040] 04 08 12 ei SM cur InOe 2 eee ia ua Cu ent meu cry 0+ &F cos. ar To find the maximum value for r we find $2, whichis e” e for rwefind Then e*sind + efcos| 0. Since e? £0, we know sin@ = ~cos8, 0. So e°(sind + cosé) ‘This means tand =~, and @= Tow 84 Sin, of ar ar Note that not all curves will give a useful result for ©. The curve r=@ has <"= 1, which not all curves will 2 ul result for $F 7 implies the distance of the curve from the origin is always increasing. Mrcientee) fhe maximum distance of the curve r= @e~* from the origin. ‘Sketch the curve first to see the approximate location of the maximum value, (A sketch is not always necessary) Note: This curve is not looped but does have a maximum distance. Differentiate the function, Set equal to zero, noting that e-* #0. Determine the maximum distance. Since these curves also lie in the Cartesian plane, we can consider the maximum and minimum values of x and y. To do this, we must go back to the original parametric equations for polar curves. Recall from Section 5.1 that x= {()cos@ and y= (0) siné. To determine maxima and ey minima we simply ned to find 2 or +0080, We want to determine the Let us look at a curve that we have seen before: r= ‘maximum value for y. re eC ny (025 0180 0.75 1.00 1.35 1.50 1.75 apo 050 — 1 + c0s"0 + costo Soit y 2c0s*@ + 008.4 1. This simplifies to (2cosd~ 1)(cos0+ 1) dy 2 2 1 + ¢0s6)sing, then £7 = cosd ~ sin? + c0s*9 From ie dat nent 02S, Soforthe maximum ye wouse = 8° rau (142) 8233 : a 2/2 4 Note that using, =m we get y=0, since at 0= we are at the cusp of the curve, Prrcianuateee Find the minimum y value for r= 1+ sind. 06 07 08 09 ei Cenc IOS 7 ea ua Car en ae LUC eg y=(1+sind)sind Write yin a simplified form sind + sin®@ = cos +2sin cos Differentiate and put equal to 0. cos (1 +2sing) ‘1 3x Ux Identify all possible values, ~ Determine the correct value for minimum y. Finally, consider the curve r= sin26 for which we want to maximise x. Starting with >= 26 030, this gives “X= 2cos 28.056 — sin28 sind ao Using double angle formulae will lead to 2080 (30s? 4 - 2)=0. Viton inom tence, sme = 2x (yf?) =F v3 v3 9 bah t 03 earned Boat) Oi @2 oo eo oO: @-« o@ Find the area contained in the curve r= @ between ont and 0=5. cos — sind from the origin, and the coordinates at which this Find the maximum distance of the curve happens. Given that r= 1+ 6080, show that the maximum value of yis °¥3 Without using a calculator, find the area bounded by the curve and the lines @=1 and @=2. Thecurve r= V{In) is defined for 5 < 0 < x. Find the area bounded by the curve over this interval, giving oos(@ The diagram shows the curves r= 1+ cosé and r= 1 + sind, Without using a calculator, find the area of the your answer correct to 3 significant figures. Find the maximum distance of the curve ) from the origin for the interval 0 <0-< shaded region, giving your answer in an exact form, A polar curve is given as r= cos20 + cos6, a Show that x= cos@ + cos? ~ 2cos"@. » Differentiate the result in part a and show that for stationary points sin6(6cos?@ + cos@ ~ 1) = 0. € Deduce the minimum value of x. Without using a calculator, find the area enclosed by the curve r= 6" and the lines @= 1 and @=2. aa Cambridge International AS &A Level Further Mathematics: Further Pure Mathematics 1 ec Ei Peas) for 0<0< be Draw a sketch of the curve C whose polar equation is r On the sai iagram draw the line @ avers O 2. 42;,8 P0214 25+ 8 @ gait ‘Therefore, the shortest distance from P to the line is rennet Find the shortest distance between the point P(2, 1,4) and the line radi tdj+ Sk+Gi+)+ Wr Answer OP = 2145 +4k State position of point. OG = (4401+ (44 0)+6+0k State a general point on the line. SPO=(2+Hi+(3+9i+(1 +k Determine the vector PG in terms of 7. Let line direction be b =i+j +k ‘Dot vectors and determine the value of POb=0364+3=0 Make a sketch of this ag ‘ype of problem. Draw = any vector through 2 PO Find PQ and hence the shortest distance. point P, then construct >Fol=v2 a vector triangle from An alternative method tothe one shown in Worked example 65 is to make us ofthe veetor product If we fist consider the vector AP, then the shortest distance from P to the line is |ABhin& Next, multiply by the magnitude ofthe unit vetor al. This does not change the distance Now thedlatanesis iFipisan ox (ioe, miuy the vem pn ost : 1 this example, the unit vector is w= vP4Pe? (+54). The magnitude of ++ kis va /3. Given A(4,4, 5) and PQ,1,4), AP = ~3)-k. fair igca ca) 21+) +1). Hence, | ‘This is the same result but takes much less work, Remember that the vector w is the unit ‘vector of the line’s direction b. Answer 40,3,-4) 2 AB = “14 2j)-k> This leads us to another useful result, Finding the shortest distance between two st lines uses a similar approach. Imagine two lines, x; = a) + byt and r= a2 + bat, passing through points P and Q, respectively. These lines are also such that a; = OS and a)= OP, and the direction PG is parallel to b, x bs. We are looking for the distance [PO}. This is equal to |STicos 0. ‘Remember that [ST] must be greater than |PG| since [PQ is the shortest distance between the ‘two lines, ST-PG _ STH, xb.) (STPOL 1S7-by x bal ay ~ ay, the distance is So if the distance is [STfeos #, note that cos @ = ‘Substituting this new form for cos @ and noting that S7* [a — a,b, xb.) bb xb ‘The outer modulus signs are used to ensure that the distance is positive i j++ G+ 2} 4 Shds and ry = 31+ 4 + (21 | + Shoe We will (a ~ a0, xb) thy bal For example, let ry = find the shortest distance between them, For the distance, we have [PQ] Calculate each part in turn. First, a, — a, =i—j+3k. ijk i245 2-15 Then b, xb, 31-11) + 3k, and |b, x by = v299, ei ue ane eee ua Cur ues ea Eco a + 3k)-(13i- 11) +3k)| _ 33299 Hence, (Pj = [21+ *# CA = T+ 259 299 ences Find the shortest distance between the lines r= 21+ 3k + (-i—J+k)s and ry =i—j + 2k + 2ke. Answer Find the vector 57. 2143 Find the erass product of the directions of the two lines. |b, xb =2v3 Find the modulus of the normal vector. 2)=0 Determine the value of (a, — a,)-(b, x b,)- Palo State the distance between the two lines. You may need to find the position vectors of the points P and Q rather than the distance between them, Consider the two lines ry =i +j+2k+(i—j+K)s and rp =2+k+G+i)-Kyr Let P be on ry such that OF = (1 + s)i+ (1 ~9j + @ +9)k, and let Q be on r, such that OG = + 01+ §+( — Hk. Then PO =(1—s+ 01+ C1 +54 o)- (1 +54 Ok. This direction is perpendicular to both direction vectors so PO-{i- i+ k) = PO4i+j-k)=0. and From these two scalar products we have 35+ ¢=1 and s +3) Solving the equations sivess=4, = —1. Hence, OF sak The points P and Q ie on the lines 1) = 21+ 3) + (-i+2)+Ws and x)= 3i+j+ i+ jt respectively, such that PG is perpendicular to both lines. Find the coordinates of P and Q, and find the distance between them. Answer OF = 2 - s+ 04201-4098 See icone el Panda, OG =3-Hi+ +0) BG = (1 +5— dit (-2-254.0)—sk Form the perpendicular vector PO. FOWi+ 21+ 8) =0,FOCi+)=0 Find the scalar product of PB with the two direction ina td hed On xectors 1-s+1-2-20+ 36s 436=5, Form equations in sand « Solve the equations to find sand 1 Cera ace} | exercise ce | 1 For each case, find the vector equation of the line through the points given. Give your answer in the form +b, a (2, 3, S)and (-7, 1, 6) b G1, Nand 5, 6,1) ¢ (0,2, 4) and (1, 1, -1) 2. Determine the equation of the line that passes through (1,7, ~2) and is perpendicular to both 21 ~ 3) + 5k and 4i—j+2k, 3 Determine the equation of the line that passes through (3, 5, 1) and is perpendicular to both —i + 2j — 4k and 31+ 2, i 4. Three points are given as A(2,2, 1), B(.0,4) and C23, 1). Find the shortest distance between the point C and the line through A and B. Give your answer in the form ©, 5 Depts Pt are cert res vn 40143) nd 9 214] ay seeped PG is perpendicular to both lines, find the postion vectors OF and 0. @®@ «& Telines r a Show that the lines do not intersect = 5k + (144) + 3K) are skew. =2j+3k+ is and r= b_ Find the angle between the lines. ¢ Find the shortest distance between the lines. © 7 Trelines 2) r=1- + (| 28s and Ly ¥= 2144+ Gi-+008 61+ 1s, where O < 0 < 2x, are skew, a Show that these lines do not intersect regardless of the value of 8 b_ Determine the shortest distance between them. @ 8 Inboth of the following cases, determine the value(s) of the unknown constants for whieh the two lines intersect. a r=ait4j+2k + (i—j—3k)s and r=4i43j—k + Qi bo r=2%+5k+ (+2) +bk)s and r= 3-24 dk + G+ pe 2j-lor Cambridge International AS & A Level Further Mathematics: Further Pure Mathematics 1 6.3 Planes In Mathematics, a plane is an infinite two-dimensional surface. It can be represented in different ways, but one property about planes is very important. Inside a plane it is clear that there are many direction vectors, but for each individual plane there is one direction that is important, the direction normal to the plane. Suppose the normal to a plane is given as n= ai + 6j + ck and we know one point A(p, 4,”) in the plane. Using a general point R(x, y,2) in the plane we can generate a vector AR in the plane, soit is perpendicular to the normal. With AR =1—a itisclear that AR-n=0. Then (r-a)s scalar equation ofa plane, ron =a, 0, which leads to the » Using the scalar product leads to ax + by + ez = d, the Cartesian equation of a plane. For example, if we consider a plane with m =i — 3) +k, and we know the point (3,~5,4) is in the plane, then (I~ 3j + k)+(i~ 5] + 4k) = 22 such that r-(i— 3} +k) = 22 is the scalar ° form and x~3y+z=22 is the Cartesian form. ‘The last form is the vector equation of a plane. Consider, relative to an origin, a position vector a ofa point in a plane, Then consider two non-parallel vectors b and c in the plane ‘with appropriate scalars s and ¢, Any point in the plane can then be defined by reastbs+er S, ‘This is an extension of the idea of the vector equation of a line. We can define a point on “o a line in vector form using a point on the line plus the direction ofthe line (its direction FA] vcctor), Similarly, we can define a plane in vector form using a point in the plane plus two direction vectors in the plane. Finding the vector product of the two direction vectors produces a common perpendicular, ‘vector that is normal to the plane. From this we can find the Cartesian equation of the plane, ax + by + ez For example, suppose we want to find the equation of the plane passing through the points A(2, 1,2), BG, 0,—1) and CU, 5,1). We can write AB =i-j-3k and AC =2i+4j—k, so the yector equation of the plane is r= 21+)-+2k + (I—J—3k)s + (21+ 4j— Woy ‘To determine the other two forms of the equation, we need to find the normal, jk 1-1-3 2 4-1 arn = (2x 13) + (1x (-5)) + 2x 6) = 33, and so F-(13i— 5) + 6k) = 33 is the scalar form. ‘The equivalent Cartesian form is 13x ~ Sy + 62 = 33. + Ok, Next Using the cross product gives n= 134-5 ESI Vectors came about from the work on the geometric representation of complex numbers in the first two decades of the 19th century. Mathematicians such as Caspar Wessel, Jean Robert Argand and Carl Friedrich Gauss were major contributors. Isage Newton worked with quantities such as force and velocity, which are vector quantities, much earlier. However, Newton never mentioned the concept of a vector. ory gn racy euted Given that the plane II contains the points 4(2, -5, 1), B(0,3,2) and C(-4, 1,1), find the equation of the plane in both vector and Cartesian form, Answer BA =21-8j-k . CA = 61-6) o.1=3j+ 2k + Qi 8]- Ks + (6i- Ge i jk Son=| 2-8 -1 6) + 36k or dividing 6-6 0 by -6 we get i+ j — 6k. ren =arn, where an = (2x 1) + ((-5) x1) + (1x (-6)) =-9. This leads to x+y ‘Two planes in space can interact in one of two way’: either they meet or they don’t meet, ‘When two planes meet there is line of intersection rather than a point. Clearly, when two planes meet there will be an angle between them. In fact, there are two angles, as you can see in the diagram on the right, % ny f ke Using the scalar product, cos @ = inet This angle @ is also one of the angles between the J > ~ ; planes. each normal is at right angles to the plane, we have @-+a= 180%, which means that, for example, we ether find the acute angle straight away or use = 180° - a. PN Another way of viewing this is shown in the following diagram. A kite is formed between the normals and the planes to show the link between the angles. A question will usually ask for the acute angle or the obtuse angle, rather than asking simply for the angle’. e 8 4 >< For example, consider the two planes x—y+32=4 and 2x+y~5z= 12. Weare going to find the acute angle between them. (=i + 3k)-(2i +) ~ 5k) vIIv30, which leads to a= 140.4", This is not the acute angle, so Start with cos: ‘Then using the scalar product -14 vilv30 = 180° ~ 140.4° = 39.6°, gives cosa = Note that if ny+m,>0 then ais acute. If nen, <0 then ais obtuse. If nym, =0 then the normals are at right angles. This implies the planes are also at right angles. ‘Cambridge International AS & A Level Further Mathematics: Further Pure Mathematics 1 WORKED EXAMPLE 6.10 Four points are given as A(2,3,1), B(-4, ~2, 5), CG,3,2) and D(0,0,4). Find the acute angle between the planes ABC and ACD. Answer AB = -Gi -5j+4k Find the three directions related to the two planes. ACai+k Si + 10} + 5k Find m= 4B x AC. my -5j-3k Find n= AC x AD. ie 10} + 54)-Gi- 55 ~ 3k) Use the scalar product. Visovas sep = — = ere Find the obtuse angle. Visovas 6 = 505° State the acute angle. When to planes intersect, their line of intersection isa line that is parallel to both of the planes. This means that the line of intersection is also perpendicular to both of the normals, Since the direction of the line is perpendicular to both normals, then b=m, xm, where bis the direction vector of the line. To find the vector equation of the line, we still need the position vector of a point on the line. So, using the example 3x +y —z=2 and x~y + 52=2, we know the two normals aren, k and n, =i—j + Sk. From these, the direction vector k of the line is b= =I |=4i- 16)-4k. 5 For the position vector a there are two methods that we can use, First, take 3x-by-2=2 and x-y+5c 2 and let 2=0. This gives 3x4 y=2 and 2, Solving these gives x=1 and y= I. Hence, the position vector of a point on the line of intersection is i — j, so the line of intersection is given as r=i-j + (i~4j~ kyr Note we have divided b by 4 to get the direction vector of the line of intersection in a simpler form, The reason for choosing, for example z = 0, is that it simplifies the algebra and it makes us aware of the fact that somewhere on the line there must be a point where z actually is 0, This is, of course, assuming that neither plane is parallel to the xy-plane. era WORKED EXAMPLE 6.11 Find the veetor equation of the line of intersection of the planes x +3) — and 2x+dy+2=6. “Answer For the second method, start by adding the equations of the two planes 3x4 y— 2 and xy +52=2 together to eliminate one variable. This gives 4x + 42 = 4, and so 1 ~ z, Substituting this into the equation for the second plane gives 1 — =~ y + Sz =1442. Next, let 2=1 such that x= 1-1 and y=—1+44 so we have () planes. 1 4 ), so this is parallel to the same ditection vector as before. Nfl (<2) «(sot isow sponte otnnetan ate 0, 1, “1 seat (4) L ml. This method allows us to create a free variable, in this case z, which ean be changed to any ‘value and hence behaves as a parameter MPLE 6.12 By setting y asa free variable, determine the equation of the line of intersection of the planes 3x—y+22=3 and xb3y-z Answer By -y-+2 2x+6y— Hence, 5x + ‘As well as planes intersecting planes, we can also have lines tersecting planes, Lines are either parallel to planes or they will intersect them,

You might also like